41. Anticoagulation Flashcards

1
Q

CA is a 66 year-old male with hypertension, renal disease and degenerative joint disease. In his younger years, CA was a football player and has lived with the pain of a hip injury for many years. He enters the hospital for elective hip replacement surgery. His creatinine clearance is 25 mL/min. The physician orders enoxaparin 30 mg SC BID for DVT prophylaxis. Choose the correct statement:

A. The dose is correct as ordered.
B. The dose should be 60 mg SC daily.
C. The dose should be 45 mg SC daily.
D. The dose should be 30 mg SC daily.
E. The patient should receive heparin for DVT prophylaxis.

A

D. The physician ordered the correct prophylactic dose of enoxaparin for a patient without significant renal disease (30 mg SC BID, or 40 mg SC daily). If the creatinine clearance is less than 30 mL/min, the dose is reduced to 30 mg SC once daily.

enoxaparin (Lovenox): use actual body weight for all. Px VTE (30mg SQ Q12H or 40mg SQdaily; CrCl <30, 30mgSQdaily)

Tx VTE and UA/NSTEMI: 1mg/kg SQ Q12H (or 1.5 mg/kg SC Qdaily for VTE inpatient treatment only; CrCl <30, 1mg/kg SQ daily)

Tx STEMI: For patients <75 y/o, 30mg IV bolus + 1mg/kg SQ dose, then 1 mg/kg SQ Q12H (max 100mg for first 2 doses only); if CrCl <30, 30mg IV bolus + 1mg/kg SQ dose, then 1mg/kg SQ daily. For patients ≥75 y/o, 0.75 mg/kg SQ Q12H (no bolus and max 75mg for first 2 doses only); if CrCl <30, give 1mg/kg SQ daily. For PCI: give 0.3mg/kg IV bolus if last dose was 8-12 hours before balloon inflation.

How well did you know this?
1
Not at all
2
3
4
5
Perfectly
2
Q

Select the correct statement concerning Pradaxa:

A. It is indicated for patients with valvular atrial fibrillation to reduce the risk of stroke.
B. It has more drug and food interactions than warfarin.
C. It causes the same amount of GI bleeding as warfarin.
D. It has an antidote.
E. It does not require blood testing to monitor for effectiveness.

A

E. Pradaxa is indicated for non-valvular atrial fibrillation and it does not require blood tests to monitor for effectiveness.Pradaxa causes more GI bleeds and does not have an antidote.

Oral Direct Thrombin (IIa) Inhibitors: directly inhibits thrombin (Factor IIa).

dabigatran (Pradaxa): Prevents more stroke than warfarin and favored in CHEST guidelines. Boxed warning: patients receiving neuraxial anesthesia (epidural, spinal) or undergoing spinal puncture are at risk of hematomas and subsequent paralysis, premature discontinuation increases risk of thrombotic events. CI: major active bleeding, patients with mechanical heart valves. SE: dyspepsia, gastritis-like symptoms, bleeding (including more GI bleeding), pregnancy (C). No antidote. Take whole capsule with full glass of water without regards to food. Once open bottle, use within 4 months.

Non-valvular AFib: 150mg BID; 75mg BID if CrCl 15-30

Tx of VTE and reduction in risk of recurrence of VTE: 150mg BID when CrCl >30; no recommendations when CrCl <30

Switching from warfarin to dabigatran: start when INR <2

Swithcing from UFH/LMWH: start at end of UFH infusion or within 2 hours of next LMWH dose

Missed dose: take dose if >6 hours from next dose

How well did you know this?
1
Not at all
2
3
4
5
Perfectly
3
Q

A 70 year-old patient has been using warfarin therapy in the hospital. She had a deep vein thrombosis (DVT) in her right lower leg. She is being discharged, and the outpatient pharmacist who is going to dispense her warfarin is checking her medication profile for drug interactions. The pharmacist notes that the patient is using medications which increase the risk of bleeding. She will counsel the patient on increased bleeding risk. Which of the following medications can increase her bleeding risk? (Select ALL that apply.)

A. Co-enzyme Q10
B. Clopidogrel
C. Amiodarone
D. Ginkgo biloba
E. Lithium

A

B, C, D. Clopidogrel, amiodarone, and ginkgo biloba can increase the risk of bleeding in patients taking warfarin.

Herbal/Natural Product Drug: Increase bleeding risk (ginkgo biloba, bromelain, danshen, dong quai, vitamin E, evening primrose oil, echinacea, high doses of fish oils, garlic, glucosamine, goldenseal, grapefruit, policosanol, willow bark, wintergreen oil). Decrease effectiveness of warfarin (alfalfa, American ginseng, green tea, coenzyme Q-10, vitamin K, St John’s Wort) “Alcohol Can Get Gina Singing” = alfalfa, coenzyme, green tea, ginseng, st. john’s.

How well did you know this?
1
Not at all
2
3
4
5
Perfectly
4
Q

Chief Complaint: “I can’t walk - it hurts so bad”
History of Present Illness: ST is a 72 y/o female who presents to the ER with left lower extremity swelling that started last week. Her leg became more swollen and painful over the past few days until she could no longer apply weight to her left leg. She reports no shortness of breath or decreased exercise tolerance.
Allergies: NKDA
Past Medical History: Hypertension, Type 2 Diabetes, and Heart Failure
Medications: Lantus 24 units at HS, Novolog 3 units TID before meals, Diovan HCT 160 mg/25 mg daily, Coreg CR 20 mg daily, Neurontin 600 mg TID, Lasix 20 mg daily
Physical Exam / Vitals:
Height: 5’4” Weight: 200 lbs
Vitals: BP: 167/92 HR: 92 RR: 16 Temp: 98.8 F O2 sat: 98% Pain: 8/10
General: Obese female with painful left lower extremity. Appears stated age.
Cardiovascular: RRR
Lungs: CTA bilaterally
Extremities: Left calf circumference > right. Swollen from ankle to knee on left. No swelling on right. Warm to touch, no lesions or infection.
Labs:
Na (mEq/L) = 142 (135 – 145) WBC (cells/mm3) = 5.7 (4 – 11 x 10^3)
K (mEq/L) = 3.7 (3.5 – 5) Hgb (g/dL) = 13.9 (13.5 – 18 male, 12 – 16 female)
Cl (mEq/L) = 99 (95 – 103) Hct (%) = 42 (38 – 50 male, 36 – 46 female)
HCO3 (mEq/L) = 27 (24 – 30) Plt (cells/mm3) = 322 (150 – 450 x 10^3)
BUN (mg/dL) = 31 (7 – 20) AST (IU/L) = 29 (10 – 40)
SCr (mg/dL) = 1.2 (0.6 – 1.3) ALT (IU/L) = 34 (10 – 40)
Glucose (mg/dL) = 202 (100 – 125) Albumin (g/dL) = 4.2 (3.5 – 5)
Ca (mg/dL) = 9.7 (8.5 – 10.5) A1C (%) = 10.1
Mg (mEq/L) = 1.7 (1.3 – 2.1) PT (sec) = 13 (10 - 13)
PO4 (mg/dL) = 2.8 (2.3 – 4.7) INR = 0.8

Tests:
Ultrasound of left lower extremity: DVT
EKG: NSR, no ST or T wave changes
Plan: Admit to medical floor for treatment of DVT and management of other chronic conditions.

Question:
The physician plans to start ST on enoxaparin, but would like to order a laboratory test to monitor efficacy of enoxaparin therapy. Which of the following could be recommended?

A. aPTT Q6H
B. INR daily
C. Peak anti-Xa, 4 hours after the dose
D. Trough anti-Xa, before the next dose
E. There is no laboratory test to monitor efficacy of enoxaparin therapy

A

C. Routine monitoring of enoxaparin therapy with anti-Xa levels is not necessary, but monitoring can be done in certain patients. If anti-Xa levels are ordered, they should be drawn 4 hours after the SC dose (peak).

Low Molecular Weight Heparin (LMWHs): works similar to heparin except that the inhibition is much greater for Factor Xa than Factor IIa. Boxed warning: patients receiving neuraxial anesthesia (epidural, spinal) or undergoing spinal puncture are at risk of hematomas and subsequent paralysis. CI: history of HIT, active major bleed, hypersensitivity to pork. SE: bleeding, anemia, increase LFTs, thrombocytopenia, hyperkalemia, injection site reactions (bruising), pregnancy (B). Monitoring (Anti-Xa levels can be used to monitor, but not routine (obtain peak anti-Xa levels 4 hours post dose). Monitor in pregnancy, mechanical valves, morbidly obese, and renal impairment. No real antidote, but protamine can be used. Do not expel air bubble.

How well did you know this?
1
Not at all
2
3
4
5
Perfectly
5
Q

What would be expected to occur if a patient on warfarin with a stable INR is started on amiodarone?

A. The INR would decrease and the patient may experience bleeding.
B. The INR would increase and the patient may clot.
C. The INR would increase and the patient may experience bleeding.
D. The INR would decrease and the patient may clot.
E. The INR would not change.

A

C. Amiodarone inhibits the metabolism of warfarin; therefore, increasing the INR and potentially causing the patient to bleed.

Warfarin is a substrate of CYP 2C9. Avoid use with tamoxifen. 2C9 inducers: aprepitant, bosentan, carbamazepine, phenobarbital, phenytoin, primidone, rifampin (large decrease in INR), licorice, St. John’s Wort. 2C9 inhibitors: amiodarone (decrease warfarin dose by 30-50%), azole antifungals (fluconazole, ketoconazole, voriconazole), capecitabine, etravirine, fluvastatin, fluvoxamine, macrolide antibiotics, metronidazole, tigecycline, TMP/SMX, zafirlukast

How well did you know this?
1
Not at all
2
3
4
5
Perfectly
6
Q

The pharmacist is counseling AT, a patient beginning warfarin therapy, on how to recognize if she may be bleeding internally or externally. Signs that she may be bleeding include the following: (Select ALL that apply.)

A. Xerostomia
B. Red or black stools
C. Epistaxis
D. Metallic taste in mouth when she brushes her teeth
E. Headaches, dizziness or weakness

A

B, C, D, E. Other signs of bleeding could include pain, swelling, or discomfort, bleeding from cuts that takes a long time to stop (usually more than 15 minutes), large bruises, menstrual bleeding or vaginal bleeding that is much heavier than normal, pink or brown urine (or dark, tarry stools) or vomiting blood or material that looks like coffee grounds. Dark tarry stools are more likely with NSAID-induced bleeding, but would be aggravated by an elevated INR.

How well did you know this?
1
Not at all
2
3
4
5
Perfectly
7
Q

Which of the following medical conditions put a patient at risk for development of a DVT?

A. Hypertension
B. Type 1 diabetes
C. Urinary tract infection
D. Influenza
E. Cancer

A

E. Cancer or chemotherapy is a known risk factor for development of DVT.

Risk factors: Major (surgery, major trauma, immobility, cancer, previous venous thromboembolism, pregnancy, estrogen-containing medications or selective estrogen-receptor modulators, erythropoiesis-stimulating agents), other (venous compression, increasing age, acute medical illness, inflammatory bowel disease, nephrotic syndrome, myeloproliferative disorders, paroxysmal nocturnal hemoglobinuria, obesity, central venous catherization, inherited/acquired thrombophilia)

How well did you know this?
1
Not at all
2
3
4
5
Perfectly
8
Q

TL was admitted to the local hospital for a knee replacement. Orthopedic surgery is considered high risk for venous thromboembolism (VTE) but TL did not receive any VTE prophylaxis. He developed a deep vein thrombosis and was discharged on warfarin. This was his first incidence of VTE. Normally, TL is thin and active. How long should he receive warfarin?

A. One month
B. Two months
C. Three months
D. Six months
E. Twelve months

A

C. This patient had an increased risk for VTE only due to the orthopedic surgery. He will need anticoagulation for three months.

VTE treatment

Provoked: treat for 3 months

Unprovoked: low-to-moderate risk (treat longer than 3 months), high bleeding risk (treat for 3 months)

2 unprovoked VTEs: treat long-term

How well did you know this?
1
Not at all
2
3
4
5
Perfectly
9
Q

In which of the following scenarios are the intravenous direct thrombin inhibitors considered the drugs of choice?

A. To provide anticoagulation in patients who have heparin-induced thrombocytopenia (HIT).
B. To provide anticoagulation in patients who had a recent intracranial hemorrhage.
C. To provide anticoagulation in patients who had heparin-induced hyperkalemia.
D. To provide better anticoagulation in ACS patients.
E. For patients who are allergic to latex.

A

A. Direct thrombin inhibitors have been very important clinically since they do not cross-react with heparin-induced thrombocytopenia (HIT) antibodies. Once HIT develops, the injectable direct thrombin inhibitors are the drugs of choice.

IV Direct Thrombin (IIa) Inhibitors: directly inhibit thrombin (Factor IIa). used in patients with a history of HIT. CI: active major bleed. SE: bleeding, anemia, hematoma, pregnancy (B). No cross reaction with HIT. Not antidote.

argatroban: drug of choice in HIT. must be protected from light during administration. decrease dose in hepatic impairment

bivalirudin (Angiomax): used in cardiac cath lab. decrease dose when CrCl <30

Both agents: HIT dosing (initial 2mcg/kg/min, titrate to target aPTT, max 10mcg/kg/min). PCI (IV bolus followed by infusion, all are weight-based)

How well did you know this?
1
Not at all
2
3
4
5
Perfectly
10
Q

AT is beginning warfarin therapy. She asks the pharmacist which foods are high in vitamin K. Which of the following foods are high in vitamin K? (Select ALL that apply.)

A. Cauliflower
B. Canola and soybean oils
C. Broccoli and brussels sprouts
D. Fish and fish oils
E. Green and black tea

A

A, B, C, E. It is important to counsel the patient to eat consistent amounts of vitamin K daily and avoid large, sudden changes in items rich in vitamin K.

How well did you know this?
1
Not at all
2
3
4
5
Perfectly
11
Q

A female patient who is pregnant has been admitted to the hospital with a DVT. The physician will begin heparin therapy. What is the mechanism of action of heparin?

A. Heparin potentiates factor V
B. Heparin potentiates factor IXa
C. Heparin potentiates factor Xa
D. Heparin potentiates antithrombin
E. Heparin inhibits clotting factors II, VII, IX, & X

A

D. Heparin exerts its effects via antithrombin.

Unfractionated Heparin (UFH): binds to antithrombin (AT) and inactivates thrombin (Factor IIa) and Factor Xa (as well as factors IXa, XIa, XIIa, and plasmin) and prevents the conversion of fibrinogen to fibrin.

Px VTE: 5000 units SQ Q8-12H

Tx VTE: 80 units/kg IV bolus (max 5000 units) followed by 18 units/kg/hr (max 1000 units/hr) infusion (actual body weight)

Tx ACS/STEMI: 60 units/kg IV bolus (max 4000 units) followed by 12 units/kg/hr (max 1000 units/hr) infusion (actual body weight)

Boxed warning: some products contain benzyl alcohol as preservative, use is CI in neonates and infants. CI: uncontrolled active bleed, severe thrombocytopenia, ICH, history of HIT, hypersensitivity to pork products. Warning: do not give IM due to hematoma risk. SE: bleeding, thrombocytopenia, heparin induced thrombocytopenia, hyperkalemia, osteoporosis, pregnancy (C). Monitoring: heparin is monitored via the aPTT (6 hours after initiation, then every 6 hours until therapeutic range of 1.5-2.5 x patient’s baseline aPTT), platelet count, Hgb, Hct, bleeding, thrombocytopenia. Antidote: protamine (1mg will reverse 100units of heparin, max dose 50mg)

How well did you know this?
1
Not at all
2
3
4
5
Perfectly
12
Q

A patient has developed a DVT and will be placed on dalteparin. Choose the correct statements concerning dalteparin:

A. Dalteparin is safe to use if a person has a history of heparin-induced thrombocytopenia.
B. Dalteparin is safe to use in a patient receiving concurrent neuraxial anesthesia.
C. Dalteparin is administered by intramuscular injection.
D. Dalteparin is monitored by anti-Xa levels but monitoring is not required in everyone.
E. Dalteparin cannot be used safely in a patient with a sulfa allergy.

A

D. Dalteparin is administered by subcutaneous (SC) injection and is contraindicated if the patient had a history of HIT.

dalteparin (Fragmin): use actual body weight for all. Px VTE: 2500-5000 units SQ daily. Tx UA/NSTEMI: 120 units/kg (max 10000 units) Q12H

How well did you know this?
1
Not at all
2
3
4
5
Perfectly
13
Q

Which of the following is the most likely adverse effect from the use of heparin?

A. Leukopenia
B. Hypercalcemia
C. Bleeding
D. Decreased cognitive function
E. Lupus like syndrome

A

C. The major side effect of heparin is bleeding.

Unfractionated Heparin (UFH): binds to antithrombin (AT) and inactivates thrombin (Factor IIa) and Factor Xa (as well as factors IXa, XIa, XIIa, and plasmin) and prevents the conversion of fibrinogen to fibrin.

Px VTE: 5000 units SQ Q8-12H

Tx VTE: 80 units/kg IV bolus (max 5000 units) followed by 18 units/kg/hr (max 1000 units/hr) infusion (actual body weight)

Tx ACS/STEMI: 60 units/kg IV bolus (max 4000 units) followed by 12 units/kg/hr (max 1000 units/hr) infusion (actual body weight)

Boxed warning: some products contain benzyl alcohol as preservative, use is CI in neonates and infants. CI: uncontrolled active bleed, severe thrombocytopenia, ICH, history of HIT, hypersensitivity to pork products. Warning: do not give IM due to hematoma risk. SE: bleeding, thrombocytopenia, heparin induced thrombocytopenia, hyperkalemia, osteoporosis, pregnancy (C). Monitoring: heparin is monitored via the aPTT (6 hours after initiation, then every 6 hours until therapeutic range of 1.5-2.5 x patient’s baseline aPTT), platelet count, Hgb, Hct, bleeding, thrombocytopenia. Antidote: protamine (1mg will reverse 100units of heparin, max dose 50mg)

How well did you know this?
1
Not at all
2
3
4
5
Perfectly
14
Q

A 42 year-old female with a heart condition is presenting to the hospital with a DVT. The medical resident wishes to give her a low molecular weight heparin (LMWH), but the older supervising physician insists on using heparin. What are advantages to the use of LMWHs over heparin? (Select ALL that apply.)

A. LMWHs are more efficacious than heparin in treating DVTs.
B. LMWHs are more cost effective than heparin.
C. LMWHs are easier to reverse in patients that experience significant bleeding.
D. LMWHs do not require monitoring in some patients.
E. LMWHs have a more consistent anticoagulation response.

A

B, D, E. Unlike heparin, LMWHs do not require monitoring in every patient. Anti-factor Xa levels can be monitored in select patients, but this is not necessary in most patients. LMWHs are more cost-effective than heparin.

How well did you know this?
1
Not at all
2
3
4
5
Perfectly
15
Q

Which of the following procedures can help reduce medication errors associated with heparin? (Select ALL that apply.)

A. Do not use the color of the syringe or packaging to verify the dose.
B. Provide inservices that review heparin safety, including the lower heparin flush concentrations to the higher treatment doses.
C. If possible, outsource the preparation of heparin flushes.
D. Make sure unit nurses prepare the heparin doses.
E. Have the pharmacist verify the heparin concentration for the patient’s indication.

A

A, B, C, E. Heparin as an anticoagulant comes in different strengths. To help avoid errors, heparin should not be stocked in unfamiliar concentrations. The concentration must be verified by the pharmacist prior to dispensing the dose. The color of the bag or syringe should not be used to verify the dose. Having the pharmacy prepare the flush syringes (rather than busy unit nurses) can help reduce errors. Buying prepared heparin syringes (outsourcing) is most preferable.

How well did you know this?
1
Not at all
2
3
4
5
Perfectly
16
Q

The pharmacist will counsel a patient on the correct self-administration technique for enoxaparin. Which of the following are correct counseling statements? (Select ALL that apply.)

A. This medication can cause you to bruise and/or bleed more easily.
B. Choose an area on the right or left side of your abdomen, but not within two inches from the belly button.
C. Do not expel the air bubble in the syringe prior to injection.
D. Store this medication in the refrigerator until just prior to each use.
E. It is best to rub the injection site after administration to ensure quick absorption.

A

A, B, C. Do not expel the air bubble in the syringe as it can cause the patient to get a subtherapeutic dose because some of the medicine will be lost (as long as the exact dose needed is the amount in the syringe). With some medications it is recommended to rub the site after injection, but not with drugs that can cause bleeding, such as this one.

How well did you know this?
1
Not at all
2
3
4
5
Perfectly
17
Q

Select the correct indication for dabigatran:

A. To provide anticoagulation in patients with acute coronary syndrome.
B. To reduce the risk of stroke and blood clots in patients with non-valvular atrial fibrillation.
C. To reduce the risk of stroke and blood clots in patients with ventricular tachycardia.
D. To provide anticoagulation in patients who had bleeding on heparin.
E. To reduce the risk of a secondary stroke in patients who have a subarachnoid hemorrhage.

A

B. Dabigatran is indicated to reduce the risk of stroke and systemic embolism in patients with non-valvular atrial fibrillation, to treat DVT/PE, and to reduce risk of recurrence of DVT/PE.

Oral Direct Thrombin (IIa) Inhibitors: directly inhibits thrombin (Factor IIa).

dabigatran (Pradaxa): Prevents more stroke than warfarin and favored in CHEST guidelines. Boxed warning: patients receiving neuraxial anesthesia (epidural, spinal) or undergoing spinal puncture are at risk of hematomas and subsequent paralysis, premature discontinuation increases risk of thrombotic events. CI: major active bleeding, patients with mechanical heart valves. SE: dyspepsia, gastritis-like symptoms, bleeding (including more GI bleeding), pregnancy (C). No antidote. Take whole capsule with full glass of water without regards to food. Once open bottle, use within 4 months.

Non-valvular AFib: 150mg BID; 75mg BID if CrCl 15-30

Tx of VTE and reduction in risk of recurrence of VTE: 150mg BID when CrCl >30; no recommendations when CrCl <30

Switching from warfarin to dabigatran: start when INR <2

Swithcing from UFH/LMWH: start at end of UFH infusion or within 2 hours of next LMWH dose

Missed dose: take dose if >6 hours from next dose

How well did you know this?
1
Not at all
2
3
4
5
Perfectly
18
Q

HY is a 58 year-old male with atrial fibrillation. He has been using warfarin for over two years and is normally well-controlled. His cardiologist recently began amiodarone and citalopram therapy with no other medication adjustments. He is admitted to the emergency room with weakness and bleeding gums. The INR is obtained and is 9.5. His hemoglobin is 8.4 g/dL. His pants are stained with blood which is coming from his rectum. Choose the correct course of action:

A. Hold warfarin x 1 dose and administer phytonadione 1 to 2.5 mg orally.
B. Omit the next 1-2 doses, monitor frequently, and resume therapy when the INR is in the therapeutic range.
C. Hold warfarin and administer phytonadione 2 mg by SC injection.
D. Hold warfarin therapy and give vitamin K 10 mg by slow IV injection along with four-factor prothrombin complex concentrate.
E. Hold warfarin therapy and give vitamin K 10 mg by IM injection and fresh frozen plasma.

A

D. With major bleeding, vitamin K 10 mg should be given by slow IV injection along with four-factor prothrombin complex concentrate, which is preferred over fresh frozen plasma.

Treating Supratherapeutic INRs:

Antidote is vitamin K: PO or IV only.

INR above range, but <4.5 = reduce or skip dose of warfarin. resume when therapeutic (may need to reduce dose)

INR of 4.5-10, without bleeding = PO vitamin K only if high bleeding risk 1-2.5mg. resume warfarin at lower dose

INR >10, without bleeding = PO vitamin K 2.5-5mg. hold warfarin

Major bleeding from warfarin (any INR) = IV vitamin K 5-10mg AND four factor prothrombin complex concentrate (PCC) due to less risks. hold warfarin therapy

How well did you know this?
1
Not at all
2
3
4
5
Perfectly
19
Q

HM is receiving a heparin drip. What is the name of the test used to monitor heparin for efficacy?

A. Potentiation factor
B. Factors IIa, VIIa, IXa and Xa test
C. Anti-XIa levels
D. International normalized ratio
E. The activated partial thromboplastin time

A

E. The activated partial thromboplastin time (aPTT) is used to monitor the effect of heparin. The aPTT is the time, in seconds, for plasma to clot. A normal aPTT is generally between 22-38 seconds. The therapeutic aPTT range is determined individually for each hospital or laboratory depending on the reagent.

Unfractionated Heparin (UFH): binds to antithrombin (AT) and inactivates thrombin (Factor IIa) and Factor Xa (as well as factors IXa, XIa, XIIa, and plasmin) and prevents the conversion of fibrinogen to fibrin.

Px VTE: 5000 units SQ Q8-12H

Tx VTE: 80 units/kg IV bolus (max 5000 units) followed by 18 units/kg/hr (max 1000 units/hr) infusion (actual body weight)

Tx ACS/STEMI: 60 units/kg IV bolus (max 4000 units) followed by 12 units/kg/hr (max 1000 units/hr) infusion (actual body weight)

Boxed warning: some products contain benzyl alcohol as preservative, use is CI in neonates and infants. CI: uncontrolled active bleed, severe thrombocytopenia, ICH, history of HIT, hypersensitivity to pork products. Warning: do not give IM due to hematoma risk. SE: bleeding, thrombocytopenia, heparin induced thrombocytopenia, hyperkalemia, osteoporosis, pregnancy (C). Monitoring: heparin is monitored via the aPTT (6 hours after initiation, then every 6 hours until therapeutic range of 1.5-2.5 x patient’s baseline aPTT), platelet count, Hgb, Hct, bleeding, thrombocytopenia. Antidote: protamine (1mg will reverse 100units of heparin, max dose 50mg)

How well did you know this?
1
Not at all
2
3
4
5
Perfectly
20
Q

Low molecular weight heparins have a boxed warning concerning this risk:

A. Stevens Johnson syndrome
B. Spinal or epidural hematoma formation
C. Pancreatitis
D. Severe bleeding
E. Acute renal failure

A

B. The risk is highest if the patient receives neuraxial anesthesia or has a spinal puncture concurrently. These hematomas may result in long-term or permanent paralysis.

Low Molecular Weight Heparin (LMWHs): works similar to heparin except that the inhibition is much greater for Factor Xa than Factor IIa. Boxed warning: patients receiving neuraxial anesthesia (epidural, spinal) or undergoing spinal puncture are at risk of hematomas and subsequent paralysis. CI: history of HIT, active major bleed, hypersensitivity to pork. SE: bleeding, anemia, increase LFTs, thrombocytopenia, hyperkalemia, injection site reactions (bruising), pregnancy (B). Monitoring (Anti-Xa levels can be used to monitor, but not routine (obtain peak anti-Xa levels 4 hours post dose). Monitor in pregnancy, mechanical valves, morbidly obese, and renal impairment. No real antidote, but protamine can be used. Do not expel air bubble.

enoxaparin (Lovenox): use actual body weight for all. Px VTE (30mg SQ Q12H or 40mg SQdaily; CrCl <30, 30mgSQdaily)

Tx VTE and UA/NSTEMI: 1mg/kg SQ Q12H (or 1.5 mg/kg SC Qdaily for VTE inpatient treatment only; CrCl <30, 1mg/kg SQ daily)

Tx STEMI: For patients <75 y/o, 30mg IV bolus + 1mg/kg SQ dose, then 1 mg/kg SQ Q12H (max 100mg for first 2 doses only); if CrCl <30, 30mg IV bolus + 1mg/kg SQ dose, then 1mg/kg SQ daily. For patients ≥75 y/o, 0.75 mg/kg SQ Q12H (no bolus and max 75mg for first 2 doses only); if CrCl <30, give 1mg/kg SQ daily. For PCI: give 0.3mg/kg IV bolus if last dose was 8-12 hours before balloon inflation.

dalteparin (Fragmin): use actual body weight for all. Px VTE: 2500-5000 units SQ daily. Tx UA/NSTEMI: 120 units/kg (max 10000 units) Q12H

How well did you know this?
1
Not at all
2
3
4
5
Perfectly
21
Q

A 25 year-old female is receiving warfarin for a DVT. Her counseling should include the following:

A. Warfarin is safe in pregnancy (Pregnancy Category A)
B. Warfarin may be safe in pregnancy (Pregnancy Category B)
C. Warfarin may be unsafe in pregnancy (Pregnancy Category C)
D. Warfarin is unsafe in pregnancy but can be used if the benefits outweigh the risks (Pregnancy Category D)
E. Warfarin is unsafe in pregnancy and cannot be used (Pregnancy Category X)

A

E. Warfarin is contraindicated in pregnancy (Pregnancy Category X) unless the patient has a mechanical heart valve (Pregnancy Category D).

warfarin (Coumadin, Jantoven): competitively inhibits the C1 subunit of the multi-unit vitamin L epoxide reductase (VKORC1) enzyme complex, thereby reducing the regeneration of vitamin K epoxide and causing depletion of active clotting factors II, VII, IX and X and protein C/S. Boxed warning: may cause major or fatal bleeding. CI: hemorrhagic tendencies (cerebrovascular hemorrhage, bacterial endocarditis, pericarditis, pericardial effusions), blood dyscrasias, pregnancy (except with mechanical heart valves at high risk for thromboembolism), uncontrolled hypertension, non-compliance, recent or potential surgery of the eye or CNS, major regional lumbar block anesthesia or traumatic surgery resulting in large, open surfaces, (pre-)eclampsia, threatened abortion. SE: bleeding, skin necrosis, purple toe syndrome, pregnancy (X), pregnancy (D) for mechanical heart valve. Monitor: INR target 2.5, range is 2-3 for most (2.5-3.5 for mechanical mitral valve or 2 mechanical heart valves). Antidote is vitamin K. S-enantiomer is 2.7-3.8 times more potent than R-enantiomer.

How well did you know this?
1
Not at all
2
3
4
5
Perfectly
22
Q

A patient is being started on Pradaxa. Choose the correct statement regarding Pradaxa:

A. Once a bottle of Pradaxa is opened, the capsules must be used within 15 days.
B. Once a bottle of Pradaxa is opened, the capsules must be used within 60 days.
C. Once a bottle of Pradaxa is opened, the capsules must be used within 90 days.
D. Once a bottle of Pradaxa is opened, the capsules must be used within 120 days.
E. If Pradaxa capsules are transferred to an amber container, they are good up to 2 months.

A

D. The 60-count bottles of Pradaxa expire 4 months after opening the bottle.

Oral Direct Thrombin (IIa) Inhibitors: directly inhibits thrombin (Factor IIa).

dabigatran (Pradaxa): Prevents more stroke than warfarin and favored in CHEST guidelines. Boxed warning: patients receiving neuraxial anesthesia (epidural, spinal) or undergoing spinal puncture are at risk of hematomas and subsequent paralysis, premature discontinuation increases risk of thrombotic events. CI: major active bleeding, patients with mechanical heart valves. SE: dyspepsia, gastritis-like symptoms, bleeding (including more GI bleeding), pregnancy (C). No antidote. Take whole capsule with full glass of water without regards to food. Once open bottle, use within 4 months.

How well did you know this?
1
Not at all
2
3
4
5
Perfectly
23
Q

JK is a 62 year-old female with chronic urinary tract infections. Several times a year, she receives a prescription for Bactrim. The physician suggested she use the antibiotic daily, but she prefers not to because she feels that she is already using too many medications. JK comes to the pharmacy today with a prescription for warfarin. She tells the pharmacist that the heart doctor found her heart was “beating funny.” The pharmacist should emphasize the following counseling to JK: (Select ALLthat apply.)

A. If you get a UTI, the antibiotic Bactrim could make your warfarin level increase.
B. When taking the warfarin and Bactrim, separate the doses by 4 hours to decrease the risk of the drug interaction.
C. The drug interaction between warfarin and Bactrim for a UTI may lead to significant bleeding.
D. Warfarin can cause the Bactrim to be ineffective in treating the UTI.
E. Make sure to inform your health care providers that you are using warfarin so they can choose alternative medications that do not cause drug-drug interactions.

A

A, C, E. Bactrim can inhibit the metabolism of warfarin putting the patient at risk for bleeding. All providers treating JK should know she is taking warfarin to prevent drug-drug interactions.

Warfarin is a substrate of CYP 2C9. Avoid use with tamoxifen. 2C9 inducers: aprepitant, bosentan, carbamazepine, phenobarbital, phenytoin, primidone, rifampin (large decrease in INR), licorice, St. John’s Wort. 2C9 inhibitors: amiodarone (decrease warfarin dose by 30-50%), azole antifungals (fluconazole, ketoconazole, voriconazole), capecitabine, etravirine, fluvastatin, fluvoxamine, macrolide antibiotics, metronidazole, tigecycline, TMP/SMX, zafirlukast

How well did you know this?
1
Not at all
2
3
4
5
Perfectly
24
Q

A patient has developed a DVT and will be placed on dalteparin. What is the correct brand name for dalteparin?

A. Apixaban
B. Fragmin
C. Arixtra
D. Pradaxa
E. Angiomax

A

B. The brand name for dalteparin is Fragmin.

apixaban (Eliquis)

Arixtra (fondaparinux)

Pradaxa (dabigatran)

Angiomax (bivalirudin)

How well did you know this?
1
Not at all
2
3
4
5
Perfectly
25
Q

Patients may use the following non-pharmacological method to reduce the risk of venous thromboembolism:

A. Increase their intake of green leafy vegetables.
B. Perform several reps of 10 deep squats daily, if the physician approves this type of exercise.
C. Consume lots of water.
D. Use intermittent pneumatic compression devices.
E. Consume more olive oil and green tea.

A

E. Graduated compression stockings (GCS) and intermittent pneumatic compression (IPC) devices are two non-pharmacologic measures used to prevent venous thromboembolism.

How well did you know this?
1
Not at all
2
3
4
5
Perfectly
26
Q

Chief Complaint: “I can’t walk - it hurts so bad”
History of Present Illness: ST is a 72 y/o female who presents to the ER with left lower extremity swelling that started last week. Her leg became more swollen and painful over the past few days until she could no longer apply weight to her left leg. She reports no shortness of breath or decreased exercise tolerance.
Allergies: NKDA
Past Medical History: Hypertension, Type 2 Diabetes, and Heart Failure
Medications: Lantus 24 units at HS, Novolog 3 units TID before meals, Diovan HCT 160 mg/25 mg daily, Coreg CR 20 mg daily, Neurontin 600 mg TID, Lasix 20 mg daily
Physical Exam / Vitals:
Height: 5’4” Weight: 200 lbs
Vitals: BP: 167/92 HR: 92 RR: 16 Temp: 98.8 F O2 sat: 98% Pain: 8/10
General: Obese female with painful left lower extremity. Appears stated age.
Cardiovascular: RRR
Lungs: CTA bilaterally
Extremities: Left calf circumference > right. Swollen from ankle to knee on left. No swelling on right. Warm to touch, no lesions or infection.
Labs:
Na (mEq/L) = 142 (135 – 145) WBC (cells/mm3) = 5.7 (4 – 11 x 10^3)
K (mEq/L) = 3.7 (3.5 – 5) Hgb (g/dL) = 13.9 (13.5 – 18 male, 12 – 16 female)
Cl (mEq/L) = 99 (95 – 103) Hct (%) = 42 (38 – 50 male, 36 – 46 female)
HCO3 (mEq/L) = 27 (24 – 30) Plt (cells/mm3) = 322 (150 – 450 x 10^3)
BUN (mg/dL) = 31 (7 – 20) AST (IU/L) = 29 (10 – 40)
SCr (mg/dL) = 1.2 (0.6 – 1.3) ALT (IU/L) = 34 (10 – 40)
Glucose (mg/dL) = 202 (100 – 125) Albumin (g/dL) = 4.2 (3.5 – 5)
Ca (mg/dL) = 9.7 (8.5 – 10.5) A1C (%) = 10.1
Mg (mEq/L) = 1.7 (1.3 – 2.1) PT (sec) = 13 (10 - 13)
PO4 (mg/dL) = 2.8 (2.3 – 4.7) INR = 0.8

Tests:
Ultrasound of left lower extremity: DVT
EKG: NSR, no ST or T wave changes
Plan: Admit to medical floor for treatment of DVT and management of other chronic conditions.

Question:
ST’s healthcare provider would like to start warfarin per the Anticoagulation Management protocol at the hospital. Which of the following is appropriate to start along with warfarin on Day #1 of therapy for the DVT?

A. Heparin 5000 units SQ Q8H
B. Xarelto 20 mg daily
C. Eliquis 2.5 mg BID
D. Iprivask 15 mg SC Q12H
E. Lovenox 90 mg SC Q12H

A

E. Warfarin should be started on the same day as a parenteral anticoagulant (LMWH or UFH) for DVT/PE. The parenteral anticoagulant must be used in a treatment dose, not a prophylactic dose.

Heparin: Tx VTE: 80 units/kg IV bolus (max 5000 units) followed by 18 units/kg/hr (max 1000 units/hr) infusion (actual body weight)

enoxaparin (Lovenox): use actual body weight for all. Px VTE (30mg SQ Q12H or 40mg SQdaily; CrCl <30, 30mgSQdaily)

Tx VTE and UA/NSTEMI: 1mg/kg SQ Q12H (or 1.5 mg/kg SC Qdaily for VTE inpatient treatment only; CrCl <30, 1mg/kg SQ daily)

How well did you know this?
1
Not at all
2
3
4
5
Perfectly
27
Q

Why is it important for hospitals to get INRs taken at about the same time in the morning?

A. The INR must be taken before antibiotics are administered that day.
B. The INR value will be inaccurate if taken later in the day due to the effect of meals.
C. Healthcare providers will have the INR value and be able to adjust that day’s warfarin dose.
D. All labs are taken in the morning per most hospital’s policies and procedures.
E. Warfarin, hence the INR, is affected by the diurnal rhythm of the body.

A

C. Warfarin is generally dosed in the late afternoon or evening. If the INR is taken earlier in the day, the warfarin dose can still be adjusted prior to administration. Or, if the INR is elevated, the warfarin can be held.

How well did you know this?
1
Not at all
2
3
4
5
Perfectly
28
Q

A pharmacist working in an inpatient medical ward of the local hospital is responsible for monitoring anticoagulation therapy. She routinely obtains laboratory parameters and adjusts the doses of low molecular weight heparins (LMWHs), as needed. In which of the following clinical situations is it appropriate to monitor the level of anticoagulation with LMWH therapy? (Select ALL that apply.)

A. Patients with a myocardial infarction
B. Significant renal impairment
C. Pregnancy
D. Mechanical heart valves
E. Extremes of body weight

A

B, C, D, E. All four clinical scenarios are appropriate for monitoring LMWH therapy.

Monitor in pregnancy, mechanical valves, morbidly obese, and renal impairment.

How well did you know this?
1
Not at all
2
3
4
5
Perfectly
29
Q

What routes of administration can heparin be given?

A. Intravenous and buccal administration
B. Intravenous and intramuscular administration
C. Intravenous, intramuscular and oral administration
D. Intravenous, intramuscular and subcutaneous administration
E. Intravenous and subcutaneous administration

A

E. Heparin is administered by IV or SC injection. Heparin is not administered by IM injection due to pain and the risk of hematoma formation. Heparin is not bioavailable via the oral route.

How well did you know this?
1
Not at all
2
3
4
5
Perfectly
30
Q

What would be expected to occur if a patient on warfarin with a stable INR is started on phenobarbital? (Select ALL that apply.)

A. The INR would increase.
B. The INR would decrease.
C. The patient may clot.
D. The patient may experience bleeding.
E. The INR would not change.

A

B, C. Phenobarbital is a strong hepatic enzyme inducer and would lower the INR and put the patient at risk for clotting.

Warfarin is a substrate of CYP 2C9. Avoid use with tamoxifen. 2C9 inducers: aprepitant, bosentan, carbamazepine, phenobarbital, phenytoin, primidone, rifampin (large decrease in INR), licorice, St. John’s Wort. 2C9 inhibitors: amiodarone (decrease warfarin dose by 30-50%), azole antifungals (fluconazole, ketoconazole, voriconazole), capecitabine, etravirine, fluvastatin, fluvoxamine, macrolide antibiotics, metronidazole, tigecycline, TMP/SMX, zafirlukast

How well did you know this?
1
Not at all
2
3
4
5
Perfectly
31
Q

Rivaroxaban works by the following mechanism of action:

A. Vitamin K antagonist
B. Factor Xa inhibitor
C. Direct Factor IIa inhibitor
D. PAR-1 inhibitor
E. Inhibits antithombin

A

B. Rivaroxaban is an oral Factor Xa inhibitor.

Oral Factor Xa Inhibitors (no not use in mechanical heart valve)

rivaroxaban (Xarelto): Boxed warning: patients receiving neuraxial anesthesia (epidural, spinal) or undergoing spinal puncture are at risk of hematomas and subsequent paralysis, premature discontinuation increases risk of thrombotic events. CI: active major bleeding. Warnings: avoid with moderate-severe hepatic impairment or any degree associated with coagulopathy, avoid use in severe renal impairment (DVT tx/px CrCl <30; AFib CrCl<15). SE: bleeding anemia, pregnancy (C). No antidote. Discontinue 24 hours prior to elective surgery. *Any dose ≥15mg needs to be taken with food*

Non-valvular AFib: CrCl >50 = 20mg PO daily with evening meal, CrCl 15-50 = 15mg PO daily with evening meal, CrCl <15 = avoid use

Tx VTE: 15mg PO BID with food x 21 days, then 20mg PO daily with food. CrCl <30 = avoid use

Px DVT (after hip/knee replacement): 10mg PO daily without regards to meal. CrCl <30 = avoid use

Reduction in risk of recurrence of DVT/PE: 20mg PO daily with food. CrCl <30 = avoid use

Switching from warfarin to rivaroxaban: start when INR <3

Switching from UFH/LMWH: start at end of infusion or within 2 hours of next evening LMWH dose

Missed doses: take as soon as possible on same day. Do not double up EXCEPT when taking 15mg BID (be sure to take 30mg total daily)

How well did you know this?
1
Not at all
2
3
4
5
Perfectly
32
Q

Chief Complaint: “I can’t walk - it hurts so bad”
History of Present Illness: ST is a 72 y/o female who presents to the ER with left lower extremity swelling that started last week. Her leg became more swollen and painful over the past few days until she could no longer apply weight to her left leg. She reports no shortness of breath or decreased exercise tolerance.
Allergies: NKDA
Past Medical History: Hypertension, Type 2 Diabetes, and Heart Failure
Medications: Lantus 24 units at HS, Novolog 3 units TID before meals, Diovan HCT 160 mg/25 mg daily, Coreg CR 20 mg daily, Neurontin 600 mg TID, Lasix 20 mg daily
Physical Exam / Vitals:
Height: 5’4” Weight: 200 lbs
Vitals: BP: 167/92 HR: 92 RR: 16 Temp: 98.8 F O2 sat: 98% Pain: 8/10
General: Obese female with painful left lower extremity. Appears stated age.
Cardiovascular: RRR
Lungs: CTA bilaterally
Extremities: Left calf circumference > right. Swollen from ankle to knee on left. No swelling on right. Warm to touch, no lesions or infection.
Labs:
Na (mEq/L) = 142 (135 – 145) WBC (cells/mm3) = 5.7 (4 – 11 x 10^3)
K (mEq/L) = 3.7 (3.5 – 5) Hgb (g/dL) = 13.9 (13.5 – 18 male, 12 – 16 female)
Cl (mEq/L) = 99 (95 – 103) Hct (%) = 42 (38 – 50 male, 36 – 46 female)
HCO3 (mEq/L) = 27 (24 – 30) Plt (cells/mm3) = 322 (150 – 450 x 10^3)
BUN (mg/dL) = 31 (7 – 20) AST (IU/L) = 29 (10 – 40)
SCr (mg/dL) = 1.2 (0.6 – 1.3) ALT (IU/L) = 34 (10 – 40)
Glucose (mg/dL) = 202 (100 – 125) Albumin (g/dL) = 4.2 (3.5 – 5)
Ca (mg/dL) = 9.7 (8.5 – 10.5) A1C (%) = 10.1
Mg (mEq/L) = 1.7 (1.3 – 2.1) PT (sec) = 13 (10 - 13)
PO4 (mg/dL) = 2.8 (2.3 – 4.7) INR = 0.8

Tests:
Ultrasound of left lower extremity: DVT
EKG: NSR, no ST or T wave changes
Plan: Admit to medical floor for treatment of DVT and management of other chronic conditions.

Question:
On Day #6 of the hospitalization, ST is doing better. Her chronic conditions are under better control. ST mentions to the case manager that she lives almost 2 hours from town and has no transportation to come to the clinic for labs or office visits. The physician would like to discharge ST on an oral anticoagulant that does not require laboratory monitoring. Which of the following are options for ST? (Select ALL that apply.)

A. Lovenox
B. Pradaxa
C. Arixtra
D. Xarelto
E. Eliquis

A

B, D, E. Pradaxa, Xarelto and Eliquis are oral anticoagulant options for this patient that do not require laboratory monitoring for efficacy.

How well did you know this?
1
Not at all
2
3
4
5
Perfectly
33
Q

Vitamin K given IV has a risk of the following adverse reaction:

A. Acute dystrophy
B. Seizures
C. Peripheral neuropathy
D. Anaphylaxis
E. Neuroleptic malignant syndrome

A

D. The administration of vitamin K intravenously is associated with the risk of anaphylaxis (dyspnea, cardiac arrest, hypotension, shock).

How well did you know this?
1
Not at all
2
3
4
5
Perfectly
34
Q

JW arrives at the clinic pharmacy with a prescription for apixaban 5 mg BID. He is 52 years old, 6’1” and weighs 214 lbs. His recent laboratory parameters include a Na+ 139 mEq/L and SCr of 1.1 mg/dL. At the clinic today, he was diagnosed with a DVT. Which of the following statements are correct regarding apixaban for this patient?

A. This is the correct dose for treatment of a DVT or PE.
B. Apixaban must be taken with food.
C. Apixaban is not approved for treating DVT.
D. Apixaban is indicated for stroke prevention in patients with prosthetic heart valves.
E. Apixaban carries a boxed warning regarding the risk of stroke in patients who discontinue therapy prematurely.

A

E. Apixaban dosing differs for each indication. It can be taken without regards to food. Discontinuation of therapy without adequate anticoagulation with an alternative agent increases the risk of stroke.

apixaban (Eliquis): Boxed warning: patients receiving neuraxial anesthesia (epidural, spinal) or undergoing spinal puncture are at risk of hematomas and subsequent paralysis, premature discontinuation increases risk of thrombotic events. CI: active major bleeding. Warning: use not recommended with prosthetic heart valves or severe hepatic impairment. SE: bleeding, anemia, pregnancy (B). No antidote. Discontinue 48 hours prior to elective surgery with moderate/high bleeding risk or 24 hours prior if low bleeding risk.

Non-valvular AFib: 5mg BID, or 2.5mg BID if they have at least 2 of the following: age ≥80 y/o, body weight ≤60kg, or SCr ≥1.5

Tx of VTE: 10mg PO BID x 7 days, then 5mg PO BID

Px DVT (after hip/knee replacement): 2.5mg PO BID

Reduction in risk of recurrence of DVT/PE: 2.5mg PO BID

Switching from warfarin to apixaban: start when INR <2

Switching from apixaban to anticoagulants other than warfarin: discontinue apixaban and start other anticoagulant at next scheduled dose.

Missed dose: take as soon as possible on same day; do not double up

How well did you know this?
1
Not at all
2
3
4
5
Perfectly
35
Q

Which of the following should be discussed with a patient receiving a new prescription for Pradaxa?

A. The generic name of this medication is rivaroxaban.
B. This medication must be kept in the original container. Do not put into a pill box.
C. This medication requires periodic laboratory monitoring.
D. This medication is used to prevent blood clots around your artificial heart valve.
E. Take this medication with food.

A

B. Dabigatran (Pradaxa) must be kept in the original container and discarded 4 months after opening the original container. Capsules must be swallowed whole.

Oral Direct Thrombin (IIa) Inhibitors: directly inhibits thrombin (Factor IIa).

dabigatran (Pradaxa): Prevents more stroke than warfarin and favored in CHEST guidelines. Boxed warning: patients receiving neuraxial anesthesia (epidural, spinal) or undergoing spinal puncture are at risk of hematomas and subsequent paralysis, premature discontinuation increases risk of thrombotic events. CI: major active bleeding, patients with mechanical heart valves. SE: dyspepsia, gastritis-like symptoms, bleeding (including more GI bleeding), pregnancy (C). No antidote. Take whole capsule with full glass of water without regards to food. Once open bottle, use within 4 months.

Non-valvular AFib: 150mg BID; 75mg BID if CrCl 15-30

Tx of VTE and reduction in risk of recurrence of VTE: 150mg BID when CrCl >30; no recommendations when CrCl <30

Switching from warfarin to dabigatran: start when INR <2

Swithcing from UFH/LMWH: start at end of UFH infusion or within 2 hours of next LMWH dose

Missed dose: take dose if >6 hours from next dose

How well did you know this?
1
Not at all
2
3
4
5
Perfectly
36
Q

The anticoagulation service routinely performs an audit of the charts for patients who were treated with warfarin and enoxaparin to verify that enoxaparin was bridged and discontinued correctly. BH is a 56 year old male patient who was admitted for a DVT. His laboratory and medication administration history are included below:

Day INR Medications
1 0.8 Warfarin and enoxaparin 110 mg SC Q12H
2 1.3 Warfarin and enoxaparin 110 mg SC Q12H
3 1.7 Warfarin and enoxaparin 110 mg SC Q12H
4 2.1 Warfarin and enoxaparin 110 mg SC Q12H
5 2.4 Warfarin and enoxaparin 110 mg SC Q12H
6 2.6 Warfarin and enoxaparin 110 mg SC Q12H
7 2.5 Discharge
On which day of BH’s hospitalization should enoxaparin have been discontinued?

A. Day 3
B. Day 4
C. Day 5
D. Day 6
E. Day 7

A

C. Both anticoagulants should be continued for a minimum of 5 days and until the INR is ≥ 2 for at least 24 hours. This is a common arena for quality improvement and cost saving in hospitals. Continuing the parenteral anticoagulant longer than needed increases cost, length of stay, and potential for adverse effects.

How well did you know this?
1
Not at all
2
3
4
5
Perfectly
37
Q

Which of the following is a possible side effect from the long-term use of heparin therapy?

A. Gingival hyperplasia
B. Osteoporosis
C. GERD
D. Hair growth
E. Hypokalemia

A

B. Osteoporosis can occur with long-term use. Women who are pregnant and are using heparin long-term are at risk for decreased bone density.

Boxed warning: some products contain benzyl alcohol as preservative, use is CI in neonates and infants. CI: uncontrolled active bleed, severe thrombocytopenia, ICH, history of HIT, hypersensitivity to pork products. Warning: do not give IM due to hematoma risk. SE: bleeding, thrombocytopenia, heparin induced thrombocytopenia, hyperkalemia, osteoporosis, pregnancy (C). Monitoring: heparin is monitored via the aPTT (6 hours after initiation, then every 6 hours until therapeutic range of 1.5-2.5 x patient’s baseline aPTT), platelet count, Hgb, Hct, bleeding, thrombocytopenia. Antidote: protamine (1mg will reverse 100units of heparin, max dose 50mg)

How well did you know this?
1
Not at all
2
3
4
5
Perfectly
38
Q

A hospitalized patient developed a pulmonary embolism and was started on enoxaparin therapy. The physician began warfarin therapy on Monday and wrote an order to discontinue the enoxaparin therapy the following day. The pharmacist contacted the prescriber to recommend the following action:

A. Continue the enoxaparin until the INR reaches the therapeutic range for one value.
B. Continue the enoxaparin until the INR has been therapeutic for at least 24 hours.
C. Continue the enoxaparin until the INR has been therapeutic for at least 48 hours.
D. Continue the enoxaparin for a full 7 days and the patient has been therapeutic for at least 2 of those days.
E. Pulmonary emboli cannot be treated with warfarin; the warfarin should be discontinued.

A

B. Continuation of the parenteral anticoagulant should occur for a minimum of 5 days and until the INR is therapeutic (INR at 2.0 or above in this scenario) for at least 24 hours.

How well did you know this?
1
Not at all
2
3
4
5
Perfectly
39
Q

Which of the following groups of laboratory parameters need to be monitored during heparin therapy?

A. Hematocrit, hemogloblin, platelets, and PT
B. Hematocrit, hemoglobin, platelets, AST, and ALT
C. SCr, platelets, aPTT, and PT
D. Hematocrit, hemoglobin, platelets, and aPTT
E. Platelets, aPTT, PT, and SCr

A

D. Hematocrit, hemoglobin, platelets, and aPTT are important laboratory parameters to monitor while a patient is receiving heparin therapy.

How well did you know this?
1
Not at all
2
3
4
5
Perfectly
40
Q

What is the purpose of using a heparin “lock-flush,” such as HepFlush?

A. To provide systemic anticoagulation prophylaxis
B. To provide systemic anticoagulation treatment
C. To keep IV lines open
D. To prevent HIT
E. To dilute other medications going through the same IV line

A

C. Heparin “lock-flushes” (HepFlush) are used to keep IV lines open (patent). They are not used for anticoagulation. There have been fatal errors made by choosing the incorrect heparin strength. Using a higher dose to flush a line could cause significant bleeding, including fatal hemorrhage. Many of the dosing errors have occurred in neonates.

How well did you know this?
1
Not at all
2
3
4
5
Perfectly
41
Q

Select the correct mechanism of action for Lovenox:

A. Oral direct thrombin inhibitor
B. Injectable direct thrombin inhibitor
C. Vitamin K antagonist
D. Inhibits Factor Xa and Factor IIa via antithrombin
E. Selectively inhibits Factor Xa

A

D. Lovenox is a low molecular weight heparin (LMWH). LMWHs work by binding to antithrombin which then inhibit clotting factors Xa and IIa.

Low Molecular Weight Heparin (LMWHs): works similar to heparin except that the inhibition is much greater for Factor Xa than Factor IIa. Boxed warning: patients receiving neuraxial anesthesia (epidural, spinal) or undergoing spinal puncture are at risk of hematomas and subsequent paralysis. CI: history of HIT, active major bleed, hypersensitivity to pork. SE: bleeding, anemia, increase LFTs, thrombocytopenia, hyperkalemia, injection site reactions (bruising), pregnancy (B). Monitoring (Anti-Xa levels can be used to monitor, but not routine (obtain peak anti-Xa levels 4 hours post dose). Monitor in pregnancy, mechanical valves, morbidly obese, and renal impairment. No real antidote, but protamine can be used. Do not expel air bubble.

How well did you know this?
1
Not at all
2
3
4
5
Perfectly
42
Q

When heparin is administered, the following laboratory value must be carefully monitored:

A. White blood cells
B. Eosinophils
C. Platelets
D. Amylase
E. Sodium

A

C. A serious adverse effect associated with heparin therapy is heparin-induced thrombocytopenia, or HIT. This is a significant drop in platelets caused by an immune response against platelets. Platelets must be monitored during therapy.

Heparin-induced Thrombocytopenia (HIT)

IgG mediated drug reaction that forms heparin antibodies which binds to platelet factor 4 (PF4) and heparin PF4/heparin complex. This complex binds to Fc receptors on platelets and activates platelets. HIT is defined as a platelet drop of >50% from baseline, putting patients into a prothrombotic state which could lead to HITT (heparin-induced thrombocytopenia thrombosis). HITT is when clots form in both the venous and arterial side (basically clots can form anywhere vs VTE where clots only found in venous system)

HIT treatment: stop all forms of heparin and LMWH (including heparin flushes). Stop warfarin and reverse it. Use a direct thrombin inhibitor such as argatroban. Restart warfarin at a lower dose (5mg or less) and platelets have recovered to at least 150,000/mm3

How well did you know this?
1
Not at all
2
3
4
5
Perfectly
43
Q

A patient has developed heparin-induced thrombocytopenia (HIT). He requires anticoagulation therapy for a pulmonary embolism. Which of the following agents would not pose a risk for HIT in this patient?

A. Enoxaparin
B. Argatroban
C. Dalteparin
D. Heparin
E. None of the above

A

B. Argatroban is FDA approved for use in HIT and is recommended first-line by the CHEST guidelines.

IV Direct Thrombin (IIa) Inhibitors: directly inhibit thrombin (Factor IIa). used in patients with a history of HIT. CI: active major bleed. SE: bleeding, anemia, hematoma, pregnancy (B). No cross reaction with HIT. Not antidote.

argatroban: drug of choice in HIT. must be protected from light during administration. decrease dose in hepatic impairment

bivalirudin (Angiomax): used in cardiac cath lab. decrease dose when CrCl <30

Both agents: HIT dosing (initial 2mcg/kg/min, titrate to target aPTT, max 10mcg/kg/min). PCI (IV bolus followed by infusion, all are weight-based)

How well did you know this?
1
Not at all
2
3
4
5
Perfectly
44
Q

Which of the following organizations set the guidelines for the management of antithrombotics?

A. The American Society of Hypertension Physicians, published in JNC 8
B. The American Association of Clinical Endocrinologists (AACE), published in the journal AACE
C. The American College of Chest Physicians (ACCP): Evidence-Based Clinical Practice Guidelines, published in the journal CHEST
D. The American Society of Cardiology Physicians: Anticoagulation Practice Guidelines, published in the journal Coagulation
E. The American College of Surgeons (ACS): Guidelines for Anticoagulation Reversal, published in the journal Surgery

A

C. The CHEST guidelines are used for antithrombotics. There are valuable resources on the safe use of antithrombotics on the Institute for Safe Medication Practices at www.ismp.org

45
Q

What would be expected to occur if a patient on warfarin with a stable INR is started on fluconazole?

A. The INR would increase and the patient may clot.
B. The INR would increase and the patient may experience bleeding.
C. The INR would decrease and the patient may clot.
D. The INR would decrease and the patient may experience bleeding.
E. The INR would not change.

A

B. Fluconazole inhibits the metabolism of warfarin; therefore, increasing the INR and potentially causing the patient to bleed.

Warfarin is a substrate of CYP 2C9. Avoid use with tamoxifen. 2C9 inducers: aprepitant, bosentan, carbamazepine, phenobarbital, phenytoin, primidone, rifampin (large decrease in INR), licorice, St. John’s Wort. 2C9 inhibitors: amiodarone (decrease warfarin dose by 30-50%), azole antifungals (fluconazole, ketoconazole, voriconazole), capecitabine, etravirine, fluvastatin, fluvoxamine, macrolide antibiotics, metronidazole, tigecycline, TMP/SMX, zafirlukast

46
Q

Select the correct mechanism of action for Pradaxa:

A. Oral direct Factor IIa inhibitor
B. Injectable direct thrombin inhibitor
C. Vitamin K antagonist
D. Oral Factor Xa inhibitor
E. Inhibits Factor Xa and Factor IIa via antithrombin

A

A. Pradaxa is an oral direct thrombin (Factor IIa) inhibitor. Direct thrombin inhibitors (which block thrombin, as the name suggests) decrease the amount of fibrin available for clot formation.

Oral Direct Thrombin (IIa) Inhibitors: directly inhibits thrombin (Factor IIa).

dabigatran (Pradaxa): Prevents more stroke than warfarin and favored in CHEST guidelines. Boxed warning: patients receiving neuraxial anesthesia (epidural, spinal) or undergoing spinal puncture are at risk of hematomas and subsequent paralysis, premature discontinuation increases risk of thrombotic events. CI: major active bleeding, patients with mechanical heart valves. SE: dyspepsia, gastritis-like symptoms, bleeding (including more GI bleeding), pregnancy (C). No antidote. Take whole capsule with full glass of water without regards to food. Once open bottle, use within 4 months.

47
Q

A hospitalized patient is post-op day #1 after a right hip arthroplasty. The patient has a history of a previous VTE. The doctor has recommended that the patient begin warfarin with Lovenox bridge therapy but the patient responds that he does not wish to take “rat poison”. The pharmacy intern wants to explain to the patient the risks associated with not taking an anticoagulant, when indicated. The intern should explain to the patient that he is at higher risk for the following complications if he chooses not to use the warfarin: (Select ALL that apply.)

A. Deep vein thrombosis
B. Pulmonary embolism
C. Bleeding
D. Pleural effusion
E. Cardiac tamponade

A

A, B. The primary risks in this patient from not using anticoagulants or having anticoagulants at subtherapeutic levels are deep vein thrombosis (DVT) and/or pulmonary embolism (PE).

48
Q

A new patient is using enoxaparin therapy for “bridging” until her INR level is therapeutic. She brings the following over-the-counter medicines to the pharmacy window for payment: DHEA, Women’s 50+ multivitamin, Advil Migraine, coenzyme Q10 and a B-Complex vitamin. The pharmacist should offer the following advice:

A. Advil Migraine is not safe to use with warfarin; acetaminophen is safer.
B. DHEA contains vitamin K and may make the warfarin ineffective.
C. Vitamin B complexes cannot be used with warfarin.
D. Women’s 50+ multivitamin may increase the INR.
E. Willow bark may decrease the effectiveness of warfarin.

A

A. NSAIDs, like Advil Migraine, do not raise the INR, but they do increase bleeding risk by an antiplatelet effect. Willow bark can increase the bleeding risk as well.

Herbal/Natural Product Drug: Increase bleeding risk (ginkgo biloba, bromelain, danshen, dong quai, vitamin E, evening primrose oil, echinacea, high doses of fish oils, garlic, glucosamine, goldenseal, grapefruit, policosanol, willow bark, wintergreen oil). Decrease effectiveness of warfarin (alfalfa, American ginseng, green tea, coenzyme Q-10, vitamin K, St John’s Wort) “Alcohol Can Get Gina Singing” = alfalfa, coenzyme, green tea, ginseng, st. john’s.

49
Q

Which of the following statements regarding warfarin are correct? (Select ALL that apply.)

A. The antidote is protamine.
B. Warfarin blocks the activation of clotting factors II, VII, IX, and X.
C. Patients on warfarin should not eat any vitamin K containing foods.
D. Warfarin is a vitamin K antagonist.
E. Warfarin generally takes 2 days to become therapeutic.

A

B, D. Warfarin is a vitamin K antagonist. Vitamin K is required for the carboxylation of clotting factors II, VII, IX, and X. Without adequate vitamin K, the liver produces the factors-but they have reduced coagulant activity.

50
Q

What is the name of the test used to monitor warfarin efficacy and toxicity?

A. Potentiation factor
B. Factors II, VII, IX and X
C. Anti-Xa level
D. The international normalized ratio
E. The activated partial thromboplastin time

A

D. The international normalized ratio (INR) is the test used to measure the effectiveness of warfarin.

51
Q

Select the correct dosing recommendation for dabigatran for a patient with a creatinine clearance of 54 mL/min:

A. Take a 150 mg capsule twice daily, with food.
B. Take a 150 mg capsule twice daily, without food.
C. Take a 150 mg capsule twice daily, with or without food.
D. Take a 75 mg capsule twice daily, with or without food.
E. Take 150 mg capsule once daily, with food.

A

C. For patients with atrial fibrillation and CrCl 15-30 mL/min, the recommended dose is 75 mg twice daily. Dabigatran is dosed BID and it is important that patients remember to take both doses. The medication does not last long and if it is not taken appropriately, the patient is at risk of stroke. Dabigatran is taken without regard to meals.

dabigatran (Pradaxa): Prevents more stroke than warfarin and favored in CHEST guidelines. Boxed warning: patients receiving neuraxial anesthesia (epidural, spinal) or undergoing spinal puncture are at risk of hematomas and subsequent paralysis, premature discontinuation increases risk of thrombotic events. CI: major active bleeding, patients with mechanical heart valves. SE: dyspepsia, gastritis-like symptoms, bleeding (including more GI bleeding), pregnancy (C). No antidote. Take whole capsule with full glass of water without regards to food. Once open bottle, use within 4 months.

Non-valvular AFib: 150mg BID; 75mg BID if CrCl 15-30

Tx of VTE and reduction in risk of recurrence of VTE: 150mg BID when CrCl >30; no recommendations when CrCl <30

Switching from warfarin to dabigatran: start when INR <2

Swithcing from UFH/LMWH: start at end of UFH infusion or within 2 hours of next LMWH dose

Missed dose: take dose if >6 hours from next dose

52
Q

Choose the correct antidote to use in the case of a heparin overdose:

A. Protamine
B. N-acetylcysteine
C. Vitamin K
D. Flumazenil
E. Naloxone

A

A. Protamine is the antidote for heparin.

Unfractionated Heparin (UFH): binds to antithrombin (AT) and inactivates thrombin (Factor IIa) and Factor Xa (as well as factors IXa, XIa, XIIa, and plasmin) and prevents the conversion of fibrinogen to fibrin.

Px VTE: 5000 units SQ Q8-12H

Tx VTE: 80 units/kg IV bolus (max 5000 units) followed by 18 units/kg/hr (max 1000 units/hr) infusion (actual body weight)

Tx ACS/STEMI: 60 units/kg IV bolus (max 4000 units) followed by 12 units/kg/hr (max 1000 units/hr) infusion (actual body weight)

Boxed warning: some products contain benzyl alcohol as preservative, use is CI in neonates and infants. CI: uncontrolled active bleed, severe thrombocytopenia, ICH, history of HIT, hypersensitivity to pork products. Warning: do not give IM due to hematoma risk. SE: bleeding, thrombocytopenia, heparin induced thrombocytopenia, hyperkalemia, osteoporosis, pregnancy (C). Monitoring: heparin is monitored via the aPTT (6 hours after initiation, then every 6 hours until therapeutic range of 1.5-2.5 x patient’s baseline aPTT), platelet count, Hgb, Hct, bleeding, thrombocytopenia. Antidote: protamine (1mg will reverse 100units of heparin, max dose 50mg)

53
Q

MG is a 43 year-old male who is usually well-controlled on a warfarin regimen of 7.5 mg five days per week, and 5 mg two days weekly. He presents to the anticoagulation clinic to have his INR checked. He reports that he had an upper respiratory infection and the physician had given him a 10-day course of levofloxacin. He just took his last levofloxacin tablet this morning. His INR is elevated today at 3.5; the target therapeutic INR is 2-3. Choose the preferred course of action:

A. Hold the warfarin dose today; resume usual dosing regimen when INR is therapeutic and have the patient monitor for symptoms of bleeding.
B. Hold warfarin and administer phytonadione 5 mg PO x 1 now. Resume warfarin when the INR is therapeutic.
C. Hold warfarin and administer phytonadione 2 mg by SC injection. Resume warfarin when the INR is therapeutic.
D. Hold warfarin and administer phytonadione 2 mg by IM injection. Resume warfarin when the INR is therapeutic.
E. Hold warfarin and administer phytonadione 2 mg by IV injection. Resume warfarin when the INR is therapeutic.

A

A.

Treating Supratherapeutic INRs:

Antidote is vitamin K: PO or IV only.

INR above range, but <4.5 = reduce or skip dose of warfarin. resume when therapeutic (may need to reduce dose)

INR of 4.5-10, without bleeding = PO vitamin K only if high bleeding risk 1-2.5mg. resume warfarin at lower dose

INR >10, without bleeding = PO vitamin K 2.5-5mg. hold warfarin

Major bleeding from warfarin (any INR) = IV vitamin K 5-10mg AND four factor prothrombin complex concentrate (PCC) due to less risks. hold warfarin therapy

54
Q

KP is a 58 year-old female who is usually well-controlled on a warfarin regimen of 5 mg daily. She has been sick for the past week but feels better today. She ate little during her illness. KP presents to the anticoagulation clinic to have her INR checked. Her INR is elevated today at 5.8. There is no noticeable bleeding and she is a low bleeding risk. Choose the preferred course of action:

A. Hold warfarin x 1 and administer phytonadione 1 to 2.5 mg orally.
B. Omit the next few doses, monitor frequently, and resume therapy at a lower dose when the INR is in the therapeutic range.
C. Hold warfarin and administer phytonadione 2 mg by SC injection.
D. Hold warfarin and administer vitamin K 5 mg orally.
E. Hold warfarin and administer phytonadione 2 mg by IM injection.

A

B. According to the CHEST 2012 guidelines, patients with a supratherapeutic INR of 4.5 - 10 and without bleeding should not routinely receive vitamin K. Therefore, the patient should have 1 - 2 doses of warfarin held and the INR monitored. Restart warfarin at a lower dose when the INR is in the therapeutic range.

Treating Supratherapeutic INRs:

Antidote is vitamin K: PO or IV only.

INR above range, but <4.5 = reduce or skip dose of warfarin. resume when therapeutic (may need to reduce dose)

INR of 4.5-10, without bleeding = PO vitamin K only if high bleeding risk 1-2.5mg. resume warfarin at lower dose

INR >10, without bleeding = PO vitamin K 2.5-5mg. hold warfarin

Major bleeding from warfarin (any INR) = IV vitamin K 5-10mg AND four factor prothrombin complex concentrate (PCC) due to less risks. hold warfarin therapy

55
Q

Chief Complaint: “My feet hurt”
History of Present Illness: PY is a 72 y/o white male who presents to the ER with his wife. He has increasing pain in both feet over the past 3 days. The pain radiates from the bottoms of his feet, to the top of his feet, and then up his shins. He describes a burning, itching pain. It is worse at night.

Allergies: penicillin, doxycycline
Past Medical History: Non-valvular atrial fibrillation (NVAF), hypertension, type 2 diabetes, stroke (4 years ago with no residual deficits in speech or motor skills)
Medications: Fortamet 1,000 mg QAM, Xarelto 20 mg daily with dinner, Zestril 10 mg daily, hydrochlorothiazide 12.5 mg daily, Toprol XL 50 mg daily
Physical Exam / Vitals:
Height: 6’1” Weight: 199 pounds
BP: 151/75 mmHg HR: 69 BPM RR: 14 BPM Temp: 98.2°F Pain: 8/10
CV: Irregularly, irregular
Extremities: No open lesions or cellulitis, decreased sensation bilaterally

Labs:
Na (mEq/L) = 137 (135 – 145) WBC (cells/mm3) = 6.2 (4 – 11 x 10^3)
K (mEq/L) = 3.8 (3.5 – 5) Hgb (g/dL) = 14.5 (13.5 – 18 male, 12 – 16 female)
Cl (mEq/L) = 101 (95 – 103) Hct (%) = 43.4 (38 – 50 male, 36 – 46 female)
HCO3 (mEq/L) = 25 (24 – 30) Plt (cells/mm3) = 342 (150 – 450 x 10^3)
BUN (mg/dL) = 13 (7 – 20) AST (IU/L) = 25 (10 – 40)
SCr (mg/dL) = 1.0 (0.6 – 1.3) ALT (IU/L) = 12 (10 – 40)
Glucose (mg/dL) = 276 (100 – 125) Albumin (g/dL) = 3.9 (3.5 – 5)
Ca (mg/dL) = 8.9 (8.5 – 10.5) A1C (%) = 9.8
Mg (mEq/L) = 1.5 (1.3 – 2.1) PT (sec) = 11 (10 - 13)
PO4 (mg/dL) = 2.6 (2.3 – 4.7) aPTT (sec) = 26 (22 – 38)
INR = 0.9 (0 - 1.2)

Tests:
Bilateral lower extremity ultrasound: negative for DVT bilaterally
Question:
The physician questions the pharmacist about PY’s coagulation tests. PY is not experiencing any signs or symptoms of bleeding. What is the most appropriate interpretation and plan?

A. The PT is used to measure PY’s anticoagulant. Since it is in the normal range, the anticoagulant is not working.
B. The aPTT is used to measure PY’s anticoagulant. Since it is in the normal range, the anticoagulant dose should be increased.
C. INR is used to measure PY’s anticoagulant. Since it is in the normal range, the anticoagulant is not working.
D. Anti-Xa is used to measure PY’s anticoagulant. It should be ordered to determine efficacy of PY’s anticoagulation.
E. None of the above.

A

E. No monitoring of efficacy is required for Xarelto and the dose is correct for PY based on renal function and indication.

rivaroxaban (Xarelto): Boxed warning: patients receiving neuraxial anesthesia (epidural, spinal) or undergoing spinal puncture are at risk of hematomas and subsequent paralysis, premature discontinuation increases risk of thrombotic events. CI: active major bleeding. Warnings: avoid with moderate-severe hepatic impairment or any degree associated with coagulopathy, avoid use in severe renal impairment (DVT tx/px CrCl <30; AFib CrCl<15). SE: bleeding anemia, pregnancy (C). No antidote. Discontinue 24 hours prior to elective surgery. *Any dose ≥15mg needs to be taken with food*

Non-valvular AFib: CrCl >50 = 20mg PO daily with evening meal, CrCl 15-50 = 15mg PO daily with evening meal, CrCl <15 = avoid use

Tx VTE: 15mg PO BID with food x 21 days, then 20mg PO daily with food. CrCl <30 = avoid use

Px DVT (after hip/knee replacement): 10mg PO daily without regards to meal. CrCl <30 = avoid use

Reduction in risk of recurrence of DVT/PE: 20mg PO daily with food. CrCl <30 = avoid use

Switching from warfarin to rivaroxaban: start when INR <3

Switching from UFH/LMWH: start at end of infusion or within 2 hours of next evening LMWH dose

Missed doses: take as soon as possible on same day. Do not double up EXCEPT when taking 15mg BID (be sure to take 30mg total daily)

56
Q

The pharmacist is counseling AB, a patient beginning warfarin therapy. The pharmacist informs AB that if her toes become painful and/or purple in color, she must contact her doctor right away. If purple toe develops, the following action should be taken:

A. Decrease the warfarin dose
B. Increase the warfarin dose
C. Discontinue warfarin
D. Start warm compresses to the area and call your doctor
E. Change warfarin to unfractionated heparin.

A

C. Although the frequency of purple toe syndrome is low, the number of patients receiving warfarin is high. Pharmacists should maintain vigilance when counseling patients who are taking warfarin, especially those with atherosclerotic disease who have highest risk for purple toe syndrome. If present, the patient must report to the physician and warfarin will be discontinued.

warfarin (Coumadin, Jantoven): competitively inhibits the C1 subunit of the multi-unit vitamin L epoxide reductase (VKORC1) enzyme complex, thereby reducing the regeneration of vitamin K epoxide and causing depletion of active clotting factors II, VII, IX and X and protein C/S. Boxed warning: may cause major or fatal bleeding. CI: hemorrhagic tendencies (cerebrovascular hemorrhage, bacterial endocarditis, pericarditis, pericardial effusions), blood dyscrasias, pregnancy (except with mechanical heart valves at high risk for thromboembolism), uncontrolled hypertension, non-compliance, recent or potential surgery of the eye or CNS, major regional lumbar block anesthesia or traumatic surgery resulting in large, open surfaces, (pre-)eclampsia, threatened abortion. SE: bleeding, skin necrosis, purple toe syndrome, pregnancy (X), pregnancy (D) for mechanical heart valve. Monitor: INR target 2.5, range is 2-3 for most (2.5-3.5 for mechanical mitral valve or 2 mechanical heart valves). Antidote is vitamin K. S-enantiomer is 2.7-3.8 times more potent than R-enantiomer.

Dosing: Start 10mg daily for first 2 days in healthy outpatient, then adjust dose per INR values. Doses of ≤5mg may be appropriate starting dose for those who are elderly, malnourished, taking drugs which increases warfarin levels, liver disease, heart failure, or have high risk of bleeding.

57
Q

A 42 y/o female patient with no known medical conditions has just returned from a trip to Taiwan. The patient is found to have a DVT and warfarin therapy is to be initiated as an outpatient. What is the most appropriate dosing regimen to begin warfarin in this patient?

A. The patient is required to have genetic testing done first to help determine the best dose.
B. Start the warfarin at 2.5 mg for the first 1-2 days, then adjust per the INR value.
C. Start the warfarin at 10 mg for the first 1-2 days, then adjust per the INR value.
D. Start the warfarin at 20 mg for the first day, 10 mg on day 2, then adjust per the INR value.
E. Start warfarin at 2.5 mg for the first day, 5 mg on day 2, then adjust per the INR value.

A

C. Genetic testing for warfarin is not recommended, according to the CHEST 2012 updated recommendations. A starting dose of 10 mg for the first 2 doses (then adjust per INR value) is appropriate for patients who are healthy enough to begin therapy as outpatients.

warfarin (Coumadin, Jantoven): competitively inhibits the C1 subunit of the multi-unit vitamin L epoxide reductase (VKORC1) enzyme complex, thereby reducing the regeneration of vitamin K epoxide and causing depletion of active clotting factors II, VII, IX and X and protein C/S. Boxed warning: may cause major or fatal bleeding. CI: hemorrhagic tendencies (cerebrovascular hemorrhage, bacterial endocarditis, pericarditis, pericardial effusions), blood dyscrasias, pregnancy (except with mechanical heart valves at high risk for thromboembolism), uncontrolled hypertension, non-compliance, recent or potential surgery of the eye or CNS, major regional lumbar block anesthesia or traumatic surgery resulting in large, open surfaces, (pre-)eclampsia, threatened abortion. SE: bleeding, skin necrosis, purple toe syndrome, pregnancy (X), pregnancy (D) for mechanical heart valve. Monitor: INR target 2.5, range is 2-3 for most (2.5-3.5 for mechanical mitral valve or 2 mechanical heart valves). Antidote is vitamin K. S-enantiomer is 2.7-3.8 times more potent than R-enantiomer.

Dosing: Start 10mg daily for first 2 days in healthy outpatient, then adjust dose per INR values. Doses of ≤5mg may be appropriate starting dose for those who are elderly, malnourished, taking drugs which increases warfarin levels, liver disease, heart failure, or have high risk of bleeding.

58
Q

While working on the general medicine floor of the hospital, pharmacists are tasked with adjusting warfarin doses for some patients according to the hospital protocol below:
Day INR Warfarin Dose (mg)
1 Baseline 5
2 < 1.5 5
1.5-1.9 2.5
2-2.5 1
>2.5 Hold
3 < 1.5 5-10
1.5-1.9 2.5-5
2-3 0-2.5
>3 Hold
4 < 1.5 10
1.5-1.9 5-7.5
2-3 0-5
>3 Hold
A 78 y/o patient with a PE has patient has the following laboratory results and warfarin administration history:
10/6/14 - Admission Date. Baseline INR = 1.2. Start warfarin 5 mg.
10/7/14 - INR = 1.7. Warfarin 2.5 mg
10/8/14 - INR = 2.8.

Question:
What recommendation on 10/8/14 would adhere to the hospital protocol?

A. Vitamin K 2 mg PO
B. Warfarin 1 mg
C. Warfarin 5 mg
D. Warfarin 7.5 mg
E. Warfarin 10 mg

A

B. With an INR of 2.8 on Day #3 of therapy, warfarin doses of 0-2.5 mg would adhere to the hospital protocol.

59
Q

JD is a 76 y/o male with a mechanical mitral heart valve, hypertension, type 2 diabetes, and hyperlipidemia. He was recently hospitalized for an elective procedure and discharged with a therapeutic INR of 2.8. Two weeks later he comes back for follow up and his INR is 4.0. Which of the following natural products could have caused an increase in JD’s INR? (Select ALL that apply.)

A. Glucosamine
B. Grapefruit
C. Alfalfa
D. Willow bark
E. Soy

A

A, B, D. Glucosamine, grapefruit and willow bark (among others) can increase the INR. Alfalfa can decrease the INR.

Herbal/Natural Product Drug: Increase bleeding risk (ginkgo biloba, bromelain, danshen, dong quai, vitamin E, evening primrose oil, echinacea, high doses of fish oils, garlic, glucosamine, goldenseal, grapefruit, policosanol, willow bark, wintergreen oil). Decrease effectiveness of warfarin (alfalfa, American ginseng, green tea, coenzyme Q-10, vitamin K, St John’s Wort) “Alcohol Can Get Gina Singing” = alfalfa, coenzyme, green tea, ginseng, st. john’s.

60
Q

A pharmacist receives a prescription for apixaban. What is the brand name of apixaban?

A. Eliquis
B. Xarelto
C. Fragmin
D. Elavil
E. Arixtra

A

A. The brand name for apixaban is Eliquis.

Xarelto (rivaroxaban)

Fragmin (dalteparin)

Arixtra (fondaparinux)

61
Q

A pharmacy receives a prescription for warfarin 4 mg PO daily. What is the color of the warfarin tablet?

A. Peach
B. Green
C. Tan
D. Pink
E. Blue

A

E. Warfarin 4 mg comes as a blue tablet.

Warfarin colors: 1mg (pink), 2mg (lavender/purple), 2.5mg (Christmas green), 3mg (tan), 4mg (blue), 5mg (peach), 6mg (forest green/teal), 7.5mg (yellow teeth), 10mg (whiten)

62
Q

The brand name of bivalirudin is:

A. Bebulin
B. Angiomax
C. Kcentra
D.. Iprivask
E. Arixtra

A

B. The brand name of bivalirudin is Angiomax.

63
Q

KP went to an urgent care center for treatment of cellulitis. She was prescribed Septra DS PO BID x 10 days. How will theSeptra DS affect her warfarin therapy?

A. Septra DS is contraindicated with warfarin. Instruct patient not to take the Septra DS.
B. Septra DS interacts with warfarin by increasing the INR. Recommend decreasing the warfarin dose and monitor closely.
C. Septra DS interacts with warfarin by decreasing the INR. Recommend increasing the warfarin dose by 10-20% and monitor closely.
D. Septra DS interacts with warfarin by decreasing the INR. Recommend increasing the warfarin dose by 30-50% and monitor closely.
E. Septra DS does not interact with warfarin.

A

B. Sulfamethoxazole/trimethoprim (Bactrim, Septra) impairs the hepatic metabolism of warfarin. It is a strong 2C9 inhibitor, and may also involve displacement of warfarin from protein binding sites or alterations in intestinal flora. This interaction can cause the INR to increase.

Warfarin is a substrate of CYP 2C9. Avoid use with tamoxifen. 2C9 inducers: aprepitant, bosentan, carbamazepine, phenobarbital, phenytoin, primidone, rifampin (large decrease in INR), licorice, St. John’s Wort. 2C9 inhibitors: amiodarone (decrease warfarin dose by 30-50%), azole antifungals (fluconazole, ketoconazole, voriconazole), capecitabine, etravirine, fluvastatin, fluvoxamine, macrolide antibiotics, metronidazole, tigecycline, TMP/SMX, zafirlukast

64
Q

CHADS2 scoring scale assesses stroke risk in patients with:

A. Venous thromboembolism
B. Mitral valve replacement
C. Previous ischemic strokes
D. Atrial fibrillation
E. Hypertension

A

D. CHADS2 scoring system is for patients with atrial fibrillation to help guide therapy for the prevention of ischemic strokes.

65
Q

Chief Complaint: “I feel dizzy”
History of Present Illness: GH is a 84 y/o male patient with a long history of cardiovascular disease. He has been feeling dizzy when walking, but otherwise reports that he is doing well. His PCP did an ECG in the office today and GH was found to be in atrial fibrillation.
Allergies: sulfa
Past Medical History: HTN, MI x 2 (25 and 12 years ago - no stents), Heart Failure (last ECHO: EF 29%), Type 2 Diabetes, Osteoarthritis
Medications: Mobic 7.5 mg daily, Coreg 6.25 mg BID, Zestril 10 mg daily, Lasix 40 mg daily, Lantus 7 units at HS,Glucophage XR 1,000 mg daily, aspirin 81 mg daily
Physical Exam / Vitals:
Height: 5’7” Weight: 152 lbs
Vitals: BP: 145/89 HR: 98 RR: 14 Temp: 98.8 F O2 Sat: 97% Pain: 2/10
General: Elderly male accompanied by daughter
Cardiovascular: Irregularly irregular
Lungs: Crackles bilaterally L worse than R
Extremities: 1+ pitting edema bilaterally. No infection or lesions of the extremities.
Labs:
Na (mEq/L) = 135 (135 – 145) WBC (cells/mm3) = 6.3 (4 – 11 x 10^3)
K (mEq/L) = 3.5 (3.5 – 5) Hgb (g/dL) = 16.2 (13.5 – 18 male, 12 – 16 female)
Cl (mEq/L) = 100 (95 – 103) Hct (%) = 48 (38 – 50 male, 36 – 46 female)
HCO3 (mEq/L) = 26 (24 – 30) Plt (cells/mm3) = 246 (150 – 450 x 10^3)
BUN (mg/dL) = 22 (7 – 20) AST (IU/L) = 12 (10 – 40)
SCr (mg/dL) = 1.4 (0.6 – 1.3) ALT (IU/L) = 23 (10 – 40)
Glucose (mg/dL) = 188 (100 – 125) Albumin (g/dL) = 2.9 (3.5 – 5)
Ca (mg/dL) = 8.7 (8.5 – 10.5) A1C (%) = 8.7
Mg (mEq/L) = 1.9 (1.3 – 2.1) PT (sec) = 12 (10 - 13)
PO4 (mg/dL) = 2.5 (2.3 – 4.7) INR = 1

Tests:
CXR: Cardiomegaly, hazy infiltrates bilaterally
ECG: Atrial fibrillation
Plan: Refer to the hospital for management of new onset atrial fibrillation.

Question:
GH’s physician would like to initiate stroke prophylaxis with an oral agent, but is unsure about dosing and indications of the new oral anticoagulants in the elderly? Which of the following is a correct dosing recommendation for an oral agent for this patient? (Select ALL that apply.)

A. Arixtra 2.5 daily
B. Eliquis 5 mg BID
C. Xarelto 20 mg daily with the evening meal
D. Pradaxa 150 mg BID
E. Warfarin 4 mg on Day #1 and adjust per INR

A

B, D, E. The doses for warfarin, Pradaxa, and Eliquis are correct for stroke prophylaxis in GH. A renal dosage adjustment would be required for Xarelto. Fondaparinux (Arixtra) is administered via SC injection and is not indicated for stroke prophylaxis in atrial fibrillation.

Pradaxa: Non-valvular AFib: 150mg BID; 75mg BID if CrCl 15-30

Eliquis: Non-valvular AFib: 5mg BID, or 2.5mg BID if they have at least 2 of the following: age ≥80 y/o, body weight ≤60kg, or SCr ≥1.5

66
Q

JD is a 76 y/o male with a mechanical mitral heart valve, hypertension, type 2 diabetes, and hyperlipidemia. He underwent major, high-bleeding risk surgery and is now receiving post-operative care. The physician wants to restart his enoxaparin therapy due to his mechanical mitral heart valve. According to the CHEST guidelines, when is it safe to restart JD on his enoxaparin treatment regimen after his surgical procedure?

A. Restart enoxaparin the evening of the procedure if hemostasis is achieved
B. Restart enoxaparin 24 hours after the procedure if hemostasis is achieved
C. Restart enoxaparin 48-72 hours after the procedure if hemostasis is achieved
D. Do not restart enoxaparin after the procedure since his thromboembolic risk is low
E. The patient does not need enoxaparin, only warfarin

A

C. In patients who are receiving bridging anticoagulation with therapeutic dose LMWH therapy and undergoing high-bleeding risk surgery, it is recommended to resume the LMWH therapy 48 to 72 hours after surgery.

Perioperative Management of Patients on Warfarin
Stop 5 days before major surgery. If moderate-high clotting risk (mechanical valves, AFib), use bridging therapy. Stop therapeutic-dose LMWH therapy 24 hours prior to surgery. Stop UFH 4-6 hours prior to surgery.

If undergoing minor surgery (dental, dermatologic, cataract surgery), can continue warfarin (and ASA).

If high INR, can give vitamin K (2.5-5mg IV or PO)

Resume warfarin 12-24 hours after surgery, when hemostasis is achieved. Bridging therapy can resume LMWH 48-72 hours after surgery in high bleeding risk surgeries or 24 hours after surgery in low bleeding risk.

67
Q

Chief Complaint: “I feel dizzy”
History of Present Illness: GH is a 84 y/o male patient with a long history of cardiovascular disease. He has been feeling dizzy when walking, but otherwise reports that he is doing well. His PCP did an ECG in the office today and GH was found to be in atrial fibrillation.
Allergies: sulfa
Past Medical History: HTN, MI x 2 (25 and 12 years ago - no stents), Heart Failure (last ECHO: EF 29%), Type 2 Diabetes, Osteoarthritis
Medications: Mobic 7.5 mg daily, Coreg 6.25 mg BID, Zestril 10 mg daily, Lasix 40 mg daily, Lantus 7 units at HS, Glucophage XR 1,000 mg daily, aspirin 81 mg daily
Physical Exam / Vitals:
Height: 5’7” Weight: 158 lbs
Vitals: BP: 145/89 HR: 98 RR: 14 Temp: 98.8 F O2 Sat: 97% Pain: 2/10
General: Elderly male accompanied by daughter
Cardiovascular: Irregularly irregular
Lungs: Crackles bilaterally L worse than R
Extremities: 1+ pitting edema bilaterally. No infection or lesions of the extremities.
Labs:
Na (mEq/L) = 135 (135 – 145) WBC (cells/mm3) = 6.3 (4 – 11 x 10^3)
K (mEq/L) = 3.5 (3.5 – 5) Hgb (g/dL) = 16.2 (13.5 – 18 male, 12 – 16 female)
Cl (mEq/L) = 100 (95 – 103) Hct (%) = 48 (38 – 50 male, 36 – 46 female)
HCO3 (mEq/L) = 26 (24 – 30) Plt (cells/mm3) = 246 (150 – 450 x 10^3)
BUN (mg/dL) = 22 (7 – 20) AST (IU/L) = 12 (10 – 40)
SCr (mg/dL) = 1.4 (0.6 – 1.3) ALT (IU/L) = 23 (10 – 40)
Glucose (mg/dL) = 188 (100 – 125) Albumin (g/dL) = 2.9 (3.5 – 5)
Ca (mg/dL) = 8.7 (8.5 – 10.5) A1C (%) = 8.7
Mg (mEq/L) = 1.9 (1.3 – 2.1) PT (sec) = 12 (10 - 13)
PO4 (mg/dL) = 2.5 (2.3 – 4.7) INR = 1

Tests:
CXR: Cardiomegaly, hazy infiltrates bilaterally
ECG: Atrial fibrillation
Plan: Refer to the hospital for management of new onset atrial fibrillation.

Question:
What is GH’s CHA2DS2-VASc score and the corresponding anticoagulation recommendation?

A. CHA2DS2-VASc score = 4; no anticoagulation, or aspirin, or oral anticoagulation can be considered
B. CHA2DS2-VASc score = 5; no anticoagulation, or aspirin, or oral anticoagulation can be considered
C. CHA2DS2-VASc score = 5, oral anticoagulation is recommended
D. CHA2DS2-VASc score = 6; no anticoagulation, or aspirin, or oral anticoagulation can be considered
E. CHA2DS2-VASc score = 6, oral anticoagulation is recommended

A

E.

CHADS2 VAS2

CHF - 1

HTN - 1

Age ≥75 - 2

Diabetes - 1

Stroke - 2

Vascular Disease (prior MI, PAD, aortic plaque) - 1

Age 65-74 - 1

Sex (female) - 1

68
Q

Chief Complaint: “My feet hurt”
History of Present Illness: PY is a 72 y/o white male who presents to the ER with his wife. He has increasing pain in both feet over the past 3 days. The pain radiates from the bottoms of his feet, to the top of his feet, and then up his shins. He describes a burning, itching pain. It is worse at night.

Allergies: NKDA
Past Medical History: Hip replacement 2 weeks ago, hypertension, type 2 diabetes
Medications: Fortamet 1,000 mg QAM, Xarelto 10 mg daily, Zestril 10 mg daily, hydrochlorothiazide 12.5 mg daily, Toprol XL 50 mg daily
Physical Exam / Vitals:
Height: 6’1” Weight: 199 pounds
BP: 151/75 mmHg HR: 69 BPM RR: 14 BPM Temp: 98.2°F Pain: 8/10
CV: RRR
Extremities: No open lesions or cellulitis, decreased sensation bilaterally

Labs:
Na (mEq/L) = 137 (135 – 145) WBC (cells/mm3) = 6.2 (4 – 11 x 10^3)
K (mEq/L) = 3.8 (3.5 – 5) Hgb (g/dL) = 14.5 (13.5 – 18 male, 12 – 16 female)
Cl (mEq/L) = 101 (95 – 103) Hct (%) = 43.4 (38 – 50 male, 36 – 46 female)
HCO3 (mEq/L) = 25 (24 – 30) Plt (cells/mm3) = 342 (150 – 450 x 10^3)
BUN (mg/dL) = 13 (7 – 20) AST (IU/L) = 62 (10 – 40)
SCr (mg/dL) = 1.0 (0.6 – 1.3) ALT (IU/L) = 53 (10 – 40)
Glucose (mg/dL) = 276 (100 – 125) Albumin (g/dL) = 3.3 (3.5 – 5)
Ca (mg/dL) = 8.9 (8.5 – 10.5) A1C (%) = 9.8
Mg (mEq/L) = 1.5 (1.3 – 2.1) PT (sec) = 21 (10 - 13)
PO4 (mg/dL) = 2.6 (2.3 – 4.7) aPTT (sec) = 42 (22 – 38)
INR = 1.4 (0 - 1.2)

Tests:
Bilateral lower extremity ultrasound: negative for DVT bilaterally
Question:
The physician questions the pharmacist about PY’s coagulation tests. PY is not experiencing any signs or symptoms of bleeding. What is the most appropriate interpretation and/or plan?

A. The coagulation tests indicate excess anticoagulation and high risk for bleeding. Stop Xarelto and monitor for bleeding.
B. The coagulation tests indicate that PY is not sufficiently anticoagulated. Xarelto is not working. Suggest switching to apixaban.
C. The coagulation tests are outside the reference range, but not used to monitor efficacy of rivaroxaban. Continue current therapy.
D. The coagulation tests indicate excess anticoagulation. PY must be taking warfarin tablets that he has at home.
E. The coagulation tests must have been mixed up with another patient. Repeat them immediately.

A

C. Though no monitoring of efficacy is required for rivaroxaban, it can prolong the PT and aPTT. This depends greatly on lab testing procedures and reagents used (which are not standardized for rivaroxaban and other newer agents). These labs merely indicate the “presence” of rivaroxaban. There is little more we can interpret from these labs at this time. The PT/INR may be prolonged from other causes (e.g., liver disease) that should be investigated.

69
Q

CD is a 42 year-old male with type 2 diabetes, hypertension and high cholesterol. He did not practice proper foot care and developed an infected big toe. Unfortunately, the infection spread into the bone and he was admitted to the hospital and had his toe amputated. While resting in bed after the surgery, CD developed a deep vein thrombosis. His creatinine clearance is estimated at 55 mL/min and his weight is 80 kg. The physician orders enoxaparin 80 mg SC BID for DVT treatment. Choose the correct statement:

A. The dose is correct as ordered.
B. The dose should be 80 mg SC daily.
C. The dose should be 160 mg SC BID.
D. The dose should be 30 mg IV bolus followed by 80 mg SC daily.
E. The patient should receive unfractionated heparin.

A

A.

Low Molecular Weight Heparin (LMWHs): works similar to heparin except that the inhibition is much greater for Factor Xa than Factor IIa. Boxed warning: patients receiving neuraxial anesthesia (epidural, spinal) or undergoing spinal puncture are at risk of hematomas and subsequent paralysis. CI: history of HIT, active major bleed, hypersensitivity to pork. SE: bleeding, anemia, increase LFTs, thrombocytopenia, hyperkalemia, injection site reactions (bruising), pregnancy (B). Monitoring (Anti-Xa levels can be used to monitor, but not routine (obtain peak anti-Xa levels 4 hours post dose). Monitor in pregnancy, mechanical valves, morbidly obese, and renal impairment. No real antidote, but protamine can be used. Do not expel air bubble.

enoxaparin (Lovenox): use actual body weight for all. Px VTE (30mg SQ Q12H or 40mg SQdaily; CrCl <30, 30mgSQdaily)

Tx VTE and UA/NSTEMI: 1mg/kg SQ Q12H (or 1.5 mg/kg SC Qdaily for VTE inpatient treatment only; CrCl <30, 1mg/kg SQ daily)

Tx STEMI: For patients <75 y/o, 30mg IV bolus + 1mg/kg SQ dose, then 1 mg/kg SQ Q12H (max 100mg for first 2 doses only); if CrCl <30, 30mg IV bolus + 1mg/kg SQ dose, then 1mg/kg SQ daily. For patients ≥75 y/o, 0.75 mg/kg SQ Q12H (no bolus and max 75mg for first 2 doses only); if CrCl <30, give 1mg/kg SQ daily. For PCI: give 0.3mg/kg IV bolus if last dose was 8-12 hours before balloon inflation.

70
Q

JD is a 76 y/o male with a mechanical mitral heart valve, hypertension, type 2 diabetes, and hyperlipidemia. JD will be undergoing an elective major surgical procedure and he is considered high-risk for thromboembolism.
Medications:
Zestril
Mevacor
Glucotrol XL
Jantoven
Labs:
SCr 0.85 mg/dL
Weight 200 lbs
Height 5’7”
INR 2.9
What is the best peri-operative anticoagulation plan for this patient regarding his warfarin therapy?

A. Hold warfarin approximately 5 days prior to procedure. No bridging with LMWH needed.
B. Hold warfarin approximately 5 days prior to procedure. Bridge with Lovenox 40 mg IV daily.
C. Hold warfarin approximately 5 days prior to procedure. Bridge with Lovenox 90 mg SC twice daily.
D. Hold warfarin approximately 1 day prior to procedure. Bridge with Lovenox 90 mg SC BID.
E. Hold warfarin for 2 days prior to procedure. Bridge with Lovenox 110 mg SC daily.

A

C. Patients at high risk for thromboembolism should be bridged with a therapeutic dose of enoxaparin.

Perioperative Management of Patients on Warfarin
Stop 5 days before major surgery. If moderate-high clotting risk (mechanical valves, AFib), use bridging therapy. Stop therapeutic-dose LMWH therapy 24 hours prior to surgery. Stop UFH 4-6 hours prior to surgery.

If undergoing minor surgery (dental, dermatologic, cataract surgery), can continue warfarin (and ASA).

If high INR, can give vitamin K (2.5-5mg IV or PO)

Resume warfarin 12-24 hours after surgery, when hemostasis is achieved. Bridging therapy can resume LMWH 48-72 hours after surgery in high bleeding risk surgeries or 24 hours after surgery in low bleeding risk.

71
Q

A patient is being started on warfarin therapy. Which of the following scenarios would pose an increased risk of bleeding from warfarin?

A. Concurrently taking a 2B6 inhibitor
B. Concurrently taking a 2D6 inhibitor
C. Concurrently taking a 2C9 inhibitor
D. Concurrently taking a 2D6 inducer
E. Concurrently taking a 2C9 inducer

A

C. A major 2C9 inhibitor will reduce the metabolism of warfarin, increasing the risk of bleeding. A 2C9 inducer would increase warfarin metabolism, increasing the risk of clotting.

Warfarin is a substrate of CYP 2C9. Avoid use with tamoxifen. 2C9 inducers: aprepitant, bosentan, carbamazepine, phenobarbital, phenytoin, primidone, rifampin (large decrease in INR), licorice, St. John’s Wort. 2C9 inhibitors: amiodarone (decrease warfarin dose by 30-50%), azole antifungals (fluconazole, ketoconazole, voriconazole), capecitabine, etravirine, fluvastatin, fluvoxamine, macrolide antibiotics, metronidazole, tigecycline, TMP/SMX, zafirlukast

72
Q

CJ is taking warfarin 2.5 mg daily and Lovenox treatment doses. On Day 4, CJ fell on her way to the grocery store. She is now in the ED with a left leg hematoma and a few lacerations but otherwise hemodynamically stable. The ED resident calls the pharmacist-managed anticoagulation services for advice on her anticoagulation management for her DVT treatment. Her INR today is 1.8. What should the anticoagulation pharmacist recommend regarding CJ’s therapy?

A. Discontinue Lovenox. Hold warfarin for 2 days until hematoma improves.
B. Discontinue Lovenox. Continue warfarin.
C. Continue Lovenox and warfarin.
D. Admit patient to the hospital for UFH therapy.
E. Discontinue Lovenox and warfarin.

A

C. As long as the patient is hemodynamically stable with no sign or symptoms of uncontrollable bleeding, the patient should continue to receive LMWH and warfarin therapy as directed for recent DVT treatment. It would be appropriate to educate on local measures to stop bleeding from the leg hematoma or lacerations. Since warfarin is not therapeutic, the Lovenox must be continued.

73
Q

A 76 y/o male patient with non-valvular atrial fibrillation is considered high-risk for stroke. The patient’s other medical conditions include heart failure and hypertension. There is no history of smoking or alcohol consumption. Which of the following medications is appropriate for long-term stroke prevention in this patient? (Select ALL that apply).

A. Pradaxa
B. Coumadin
C. Apixaban
D. Xarelto
E. Angiomax

A

A, B, C, D. Pradaxa, Coumadin, Apixaban, and Xarelto are all approved for stroke prevention in non-valvular atrial fibrillation.

74
Q

Choose the correct indication for Xarelto:

A. Treatment of acute coronary syndrome
B. Treatment of pulmonary embolism
C. Treatment of valvular atrial fibrillation
D. Treatment of post-operative ileus
E. Treatment of intravenous catheter clots

A

B. Xarelto is indicated for the treatment of pulmonary embolism.

75
Q

Warfarin is indicated for cardioversion. When warfarin is taken for cardioversion, what is the recommended therapeutic INR range according to the CHEST guidelines?

A. 2.5-3.5
B. 2-3
C. 1.5-2.5
D. 3-4
E. 3.5-4.5

A

B. The INR range is 2-3 for cardioversion.

76
Q

TR is started on Pradaxa. He has a CrCl of 25 mL/min. Which of the following medications pose a significant interaction with Pradaxa and should be avoided?

A. Multaq
B. Aciphex
C. Lithobid
D. Avelox
E. Digox

A

A. Multaq (dronedarone), a P-glycoprotein inhibitor, should not be used with Pradaxa in patients with a CrCl 15-30 mL/min.

77
Q

A pharmacy receives a prescription for warfarin 2.5 mg PO daily. What is the color of the warfarin tablet?

A. Lavender
B. Green
C. Tan
D. Pink
E. Blue

A

B. Warfarin 2.5 mg comes as a green tablet.

Warfarin colors: 1mg (pink), 2mg (lavender/purple), 2.5mg (Christmas green), 3mg (tan), 4mg (blue), 5mg (peach), 6mg (forest green/teal), 7.5mg (yellow teeth), 10mg (whiten)

78
Q

CJ is a 44 y/o female with hyperlipidemia and hypertension. She is referred to a pharmacist-managed anticoagulation clinic with newly diagnosed, first unprovoked episode of lower left leg DVT. She is a teacher and drinks 1 glass of wine per day. Lab/vitals: Weight: 325 lbs, Height: 6’1”, Scr 0.6 mg/dL (1 month ago), Baseline INR 1.1 Which of the following is the correct enoxaparin dose for CJ?

A. Enoxaparin 220 mg SC BID
B. Enoxaparin 150 mg SC daily
C. Enoxaparin 150 mg SC Q12H
D. Enoxaparin 100 mg SC daily
E. Enoxaparin 100 mg SC Q12H

A

C.

Low Molecular Weight Heparin (LMWHs): works similar to heparin except that the inhibition is much greater for Factor Xa than Factor IIa. Boxed warning: patients receiving neuraxial anesthesia (epidural, spinal) or undergoing spinal puncture are at risk of hematomas and subsequent paralysis. CI: history of HIT, active major bleed, hypersensitivity to pork. SE: bleeding, anemia, increase LFTs, thrombocytopenia, hyperkalemia, injection site reactions (bruising), pregnancy (B). Monitoring (Anti-Xa levels can be used to monitor, but not routine (obtain peak anti-Xa levels 4 hours post dose). Monitor in pregnancy, mechanical valves, morbidly obese, and renal impairment. No real antidote, but protamine can be used. Do not expel air bubble.

enoxaparin (Lovenox): use actual body weight for all. Px VTE (30mg SQ Q12H or 40mg SQdaily; CrCl <30, 30mgSQdaily)

Tx VTE and UA/NSTEMI: 1mg/kg SQ Q12H (or 1.5 mg/kg SC Qdaily for VTE inpatient treatment only; CrCl <30, 1mg/kg SQ daily)

Tx STEMI: For patients <75 y/o, 30mg IV bolus + 1mg/kg SQ dose, then 1 mg/kg SQ Q12H (max 100mg for first 2 doses only); if CrCl <30, 30mg IV bolus + 1mg/kg SQ dose, then 1mg/kg SQ daily. For patients ≥75 y/o, 0.75 mg/kg SQ Q12H (no bolus and max 75mg for first 2 doses only); if CrCl <30, give 1mg/kg SQ daily. For PCI: give 0.3mg/kg IV bolus if last dose was 8-12 hours before balloon inflation.

79
Q

CJ finished 3 months of anticoagulation therapy for the treatment of her DVT. How should the pharmacist counsel the patient regarding discontinuation of warfarin therapy?

A. Instruct patient to flush any remaining warfarin tablets in the toilet
B. Instruct patient to slowly taper the warfarin dose; do not abruptly discontinue
C. Instruct patient to continue monitoring for any bleeding or bruising for at least 1 week
D. Instruct patient that she is prone to the side effect of purple toe syndrome upon discontinuation
E. Instruct patient to take vitamin K 2.5 mg PO x 1

A

C. Patient should continue to monitor for any bleeding or bruising for at least 1 week after discontinuation of warfarin since the effect of warfarin may last for at least a week. Patients should also properly dispose of any remaining warfarin tablets by either grinding up with coffee beans and throwing them out in the trash or disposing of them in an appropriate prescription medication disposer. Do not flush medications, except perhaps for the few indicated by the DEA. Even with these, there are alternative methods for drug disposal. The CHEST guidelines (2012) suggest abrupt discontinuation of therapy rather than a gradual tapering of the dose for discontinuation.

80
Q

What duration of anticoagulation is recommended after cardioversion for atrial fibrillation?

A. 1 week
B. 2 weeks
C. 3 weeks
D. 4 weeks
E. Patients do not require anticoagulation after cardioversion.

A

D. Regardless of the duration of atrial fibrillation or the method of cardioversion, 4 weeks of anticoagulation is recommended afterward.

81
Q

Chief Complaint: “I feel dizzy”
History of Present Illness: GH is a 84 y/o male patient with a long history of cardiovascular disease. He has been feeling dizzy when walking, but otherwise reports that he is doing well. His PCP did an ECG in the office today and GH was found to be in atrial fibrillation.
Allergies: sulfa
Past Medical History: HTN, MI x 2 (25 and 12 years ago - no stents), Heart Failure (last ECHO: EF 29%), Type 2 Diabetes, Osteoarthritis
Medications: Mobic 7.5 mg daily, Coreg 6.25 mg BID, Zestril 10 mg daily, Lasix 40 mg daily, Lantus 7 units at HS, Glucophage XR 1,000 mg daily, aspirin 81 mg daily
Physical Exam / Vitals:
Height: 5’7” Weight: 158 lbs
Vitals: BP: 145/89 HR: 98 RR: 14 Temp: 98.8 F O2 Sat: 97% Pain: 2/10
General: Elderly male accompanied by daughter
Cardiovascular: Irregularly irregular
Lungs: Crackles bilaterally L worse than R
Extremities: 1+ pitting edema bilaterally. No infection or lesions of the extremities.
Labs:
Na (mEq/L) = 135 (135 - 145) WBC (cells/mm3) = 6.3 (4 - 11 x 10^3)
K (mEq/L) = 3.5 (3.5 - 5) Hgb (g/dL) = 16.2 (13.5 - 18 male, 12 - 16 female)
Cl (mEq/L) = 100 (95 - 103) Hct (%) = 48 (38 - 50 male, 36 - 46 female)
HCO3 (mEq/L) = 26 (24 - 30) Plt (cells/mm3) = 246 (150 - 450 x 10^3)
BUN (mg/dL) = 22 (7 - 20) AST (IU/L) = 12 (10 - 40)
SCr (mg/dL) = 1.4 (0.6 - 1.3) ALT (IU/L) = 23 (10 - 40)
Glucose (mg/dL) = 188 (100 - 125) Albumin (g/dL) = 2.9 (3.5 - 5)
Ca (mg/dL) = 8.7 (8.5 - 10.5) A1C (%) = 8.7
Mg (mEq/L) = 1.9 (1.3 - 2.1) PT (sec) = 12 (10 - 13)
PO4 (mg/dL) = 2.5 (2.3 - 4.7) INR = 1

Tests:
CXR: Cardiomegaly, hazy infiltrates bilaterally
ECG: Atrial fibrillation
Plan: Refer to the hospital for management of new onset atrial fibrillation.

Question:
What is GH’s CHADS2 score and the corresponding anticoagulation recommendation?

A. CHADS2 score = 3, aspirin 75-325 mg daily
B. CHADS2 score = 3, oral anticoagulation
C. CHADS2 score = 4, aspirin 75-325 mg daily
D. CHADS2 score = 4, oral anticoagulation
E. CHADS2 score = 6, oral anticoagulation

A

D.

CHADS2

CHF - 1

HTN - 1

Age ≥75 - 1

Diabetes - 1

Stroke - 2

82
Q

AZ currently has lung cancer and was diagnosed with a DVT. AZ has normal renal function. What anticoagulation therapy would be most appropriate for this patient?

A. Enoxaparin 40 mg SC daily for 3 months
B. Enoxaparin 1.5 mg/kg SC Q12H for 1 year
C. Enoxaparin 1 mg/kg SC Q12H for at least 3 months
D. Warfarin alone for 3 months with an INR goal 2.5-3.5
E. Warfarin alone for 6 months with an INR goal 2.5-3.5

A

C. According to CHEST and ASCO guidelines, LMWH is recommended for treatment of VTE over warfarin therapy for patients with cancer.

VTE treatment

Provoked: treat for 3 months

Unprovoked: low-to-moderate risk (treat longer than 3 months), high bleeding risk (treat for 3 months)

2 unprovoked VTEs: treat long-term

83
Q

LR is hospitalized with a DVT that was discovered during his fifth day as an inpatient. He is started on dalteparin. What other medication should be initiated in this patient?

A. Argatroban
B. Heparin
C. Enoxaparin
D. Fondaparinux
E. Warfarin

A

E. Warfarin should be initiated the same day as parenteral therapy.

84
Q

A patient has used warfarin daily for many years. She calls the pharmacy because she got stuck in a snowstorm yesterday and could not return home to take her warfarin dose last night. It is now 4:00 PM the following day. She usually takes the warfarin at 6:00 PM with dinner. She asks the pharmacist if she should take yesterday’s warfarin dose now. The pharmacist should offer the following advice:

A. You can take both doses with dinner.
B. Take yesterday’s dose now and take today’s dose at the normal time.
C. Do not take yesterday’s dose; missed doses are not used on the following day.
D. Contact the physician at once for an emergency check of the INR.
E. None of the above.

A

C. Instruct patients that if they miss a dose, take the dose as soon as possible on the same day. Do not take a double dose the next day to make up for a missed dose.

85
Q

LA has been initiated on Lovenox 90 mg SC Q12H for ACS. She has received 3 doses, the last dose was given 4 hours ago. She now has significant bleeding from her IV lines. Which of the following are appropriate measures to initiate? (Select ALLthat apply.)

A. Administer phytonadione now
B. Stop Lovenox therapy
C. Administer protamine now
D. Start argatroban therapy
E. Call code blue

A

B, C. The patient is experiencing significant bleeding. Discontinue Lovenox and give protamine which will neutralize ~60% of the anti-Factor Xa activity of Lovenox.

86
Q

DV has been admitted for a PE. He started heparin and warfarin therapy 3 days ago. Today, it is suspected that he has heparin-induced thrombocytopenia (platelet count is 98,000/mm3). Which of the following statements are true regarding HIT and treatment of DV? (Select ALL that apply.)

A .Heparin should be discontinued immediately.
B. Warfarin should be discontinued.
C. HIT is defined as a platelet count < 100,000/mm3
D. Vitamin K should be given
E. Platelets should be administered

A

A, B, D. HIT is defined as a > 50% drop in platelet count from baseline.

87
Q

Which of the following counseling points for Pradaxa is correct?

A. If you miss a dose, take as soon as you remember as long as your next dose is more than 6 hours away.
B. In addition to bleeding, common side effects include paresthesias and muscle weakness.
C. If you cannot swallow the capsule, open it and put the beads in a small amount of applesauce.
D. You can use ginkgo and other herbal medications safely with this medication.
E. Your blood will need to be tested to measure the effectiveness of this medication.

A

A. Swallow the capsules whole; do not break, chew, or crush (the capsules should never be opened).

Oral Direct Thrombin (IIa) Inhibitors: directly inhibits thrombin (Factor IIa).

dabigatran (Pradaxa): Prevents more stroke than warfarin and favored in CHEST guidelines. Boxed warning: patients receiving neuraxial anesthesia (epidural, spinal) or undergoing spinal puncture are at risk of hematomas and subsequent paralysis, premature discontinuation increases risk of thrombotic events. CI: major active bleeding, patients with mechanical heart valves. SE: dyspepsia, gastritis-like symptoms, bleeding (including more GI bleeding), pregnancy (C). No antidote. Take whole capsule with full glass of water without regards to food. Once open bottle, use within 4 months.

Non-valvular AFib: 150mg BID; 75mg BID if CrCl 15-30

Tx of VTE and reduction in risk of recurrence of VTE: 150mg BID when CrCl >30; no recommendations when CrCl <30

Switching from warfarin to dabigatran: start when INR <2

Swithcing from UFH/LMWH: start at end of UFH infusion or within 2 hours of next LMWH dose

Missed dose: take dose if >6 hours from next dose

88
Q

KC is a 55 y/o female with a mechanical aortic heart valve, atrial fibrillation, and osteoporosis. What is the recommended therapeutic INR range of warfarin for this patient?

A. 3.5-4.5
B. 2.5-3.5
C. 2-3
D. 1.5-2.5
E. 3-4

A

C. According to the 2012 CHEST guidelines, patients with a mechanical aortic valve should have an INR between 2-3.

Monitor: INR target 2.5, range is 2-3 for most (2.5-3.5 for mechanical mitral valve or 2 mechanical heart valves). Antidote is vitamin K. S-enantiomer is 2.7-3.8 times more potent than R-enantiomer.

89
Q

A long distance traveler comes into the pharmacy requesting information on strategies to reduce his clotting risk. He suffered from a DVT 2 years ago and is worried it will happen again. An appropriate recommendation to reduce VTE risk is:

A. Start taking aspirin 81 mg daily.
B. Start taking aspirin 325 mg daily.
C. Obtain a prescription for rivaroxaban.
D. Obtain a prescription for warfarin.
E. Do calf muscle exercises on the plane.

A

E. Aspirin or anticoagulation should not be used in this case.

90
Q

Which of the following statements regarding Kcentra are correct? (Select ALL that apply.)

A. Kcentra can cause severe bleeding
B. Kcentra is indicated for HIT
C. Kcentra should be administered with vitamin K concurrently
D. Kcentra is contraindicated in patients with HIT
E. Kcentra contains factors II, VII, IX, X, protein C and protein S

A

C, D, E. Kcentra should not be used in HIT as it contains heparin in the formulation. Kcentra can cause clotting.

91
Q

While working on the general medicine floor of the hospital, pharmacists are tasked with adjusting warfarin doses for some patients according to the hospital protocol below:
Day INR Warfarin Dose (mg)
1 Baseline 5
2 < 1.5 5
1.5-1.9 2.5
2-2.5 1
>2.5 Hold
3 < 1.5 5-10
1.5-1.9 2.5-5
2-3 0-2.5
>3 Hold
4 < 1.5 10
1.5-1.9 5-7.5
2-3 0-5
>3 Hold
A patient has the following laboratory results and warfarin administration history:
10/2/14 - Admission Date. Baseline INR = 0.8. Start warfarin 5 mg.
10/3/14 - INR = 0.9. Warfarin 5 mg.
10/4/14 - INR = 1.3. Warfarin 10 mg.
10/5/14 - INR = 1.6.

Question:
What recommendation on 10/5/14 would adhere to the hospital protocol?

A. Hold warfarin
B. Warfarin 1 mg
C. Warfarin 2.5 mg
D. Warfarin 7.5 mg
E. Warfarin 10 mg

A

D. With an INR of 1.6 on Day #4 of therapy, a warfarin dose of 5-7.5 mg is appropriate according to the hospital protocol.

92
Q

RN is a 77 year-old female who has just been released from the hospital with atrial fibrillation. She was titrated as an inpatient to a warfarin dose of 4 mg daily. She states that she does not know why she has a heart problem. She has exercised and eaten a healthy diet her entire life. The nursing student instructed RN to stop eating anything “green” including salads, which she enjoys. Choose the correct statement:

A. She should be instructed to replace the green leafy vegetables with other options, such as cauliflower and cabbage.
B. She will have to eat more nuts and fruits and eliminate many green vegetables from her daily diet.
C. The nursing student is correct; salads and other green leafy vegetables should be avoided when using warfarin therapy.
D. She should continue her regular, healthy diet but will need to keep the amount of vitamin K-containing foods fairly consistent.
E. She will not have to follow any specific dietary precautions.

A

D. Avoiding vitamin-K rich foods is not healthy. When vitamin K is avoided completely, the INR is more difficult to manage. Consistency is key. It is especially important not to make sudden, large changes in foods rich in vitamin K.

93
Q

A 77 year old patient with atrial fibrillation, hypertension, Crohn’s disease and heart failure is seen in clinic today. Her medications include Norvasc, Entocort EC, Diovan, Coreg, and Lasix. In order to optimize stroke prevention, the following recommendation should be made to the physician overseeing her care:

A. Start this patient on aspirin therapy, based on CHADS2 score.
B. Discontinue Norvasc since this can increase the risk of stroke.
C. This patient is not a candidate for warfarin therapy due to significant drug interactions.
D. Add warfarin therapy to this patient’s regimen, based on CHADS2 score.
E. Discontinue Diovan since this can increase the risk of stroke.

A

D.

CHADS2

CHF - 1

HTN - 1

Age ≥75 - 1

Diabetes - 1

Stroke - 2

94
Q

A pharmacist receives a new prescription for Xarelto. What is the generic name of Xarelto?

A. Warfarin
B. Dabigatran
C. Rivaroxaban
D. Apixaban
E. Ticagrelor

A

C. The generic name for Xarelto is rivaroxaban.

warfarin (Coumadin, Jantoven)

dabigatran (Pradaxa)

apixaban (Eliquis)

95
Q

A patient has been using warfarin therapy for several years. He does not like the time it takes to report for lab monitoring. He asks if he can have his INR checked every six months. The ACCP guidelines recommend the following monitoring schedule for patients who are well-controlled on a stable dose of warfarin:

A. The INR should be checked at least weekly in all patients.
B. The INR can be checked at up to 12 week intervals in stable patients.
C. The INR should be checked monthly in all patients.
D. The INR should be checked at least every 4 months in stable patients.
E. The INR should be checked at least every 6 months in stable patients.

A

B. Once controlled on a stable dose of warfarin for a reasonable time period, the INR can be monitored at up to a 12 week interval.

96
Q

While working as an inpatient clinical pharmacist, one of the job duties is to make sure that patients who need prophylaxis for venous thromboembolism (VTE) are receiving it. Which of the following conditions are risk factors for developing a VTE? (Select ALL that apply.)

A. Pregnancy
B. Estrogen-containing medications
C. Obesity
D. Major trauma or surgery
E. Previous history of DVT or PE

A

A, B, C, D, E. All of the conditions can increase the risk of VTE.

Risk factors: Major (surgery, major trauma, immobility, cancer, previous venous thromboembolism, pregnancy, estrogen-containing medications or selective estrogen-receptor modulators, erythropoiesis-stimulating agents), other (venous compression, increasing age, acute medical illness, inflammatory bowel disease, nephrotic syndrome, myeloproliferative disorders, paroxysmal nocturnal hemoglobinuria, obesity, central venous catherization, inherited/acquired thrombophilia)

97
Q

Chief Complaint: “I feel dizzy”
History of Present Illness: GH is a 84 y/o male patient with a long history of cardiovascular disease. He has been feeling dizzy when walking, but otherwise reports that he is doing well. His PCP did an ECG in the office today and GH was found to be in atrial fibrillation.
Allergies: sulfa
Past Medical History: Hypertension, Myocardial Infarction x 2 (25 and 12 years ago - no stents), Heart Failure (last ECHO: EF 29%), Type 2 Diabetes, Osteoarthritis
Medications: Mobic 7.5 mg daily, Coreg 6.25 mg BID, Zestril 10 mg daily, Lasix 40 mg daily, Lantus 7 units at HS,Glucophage XR 1,000 mg daily, aspirin 81 mg daily
Physical Exam / Vitals:
Height: 5’7” Weight: 152 lbs
Vitals: BP: 145/89 HR: 98 RR: 14 Temp: 98.8 F O2 Sat: 97% Pain: 2/10
General: Elderly male accompanied by daughter
Cardiovascular: Irregularly irregular
Lungs: Crackles bilaterally L worse than R
Extremities: 1+ pitting edema bilaterally. No infection or lesions of the extremities.
Labs:
Na (mEq/L) = 135 (135 – 145) WBC (cells/mm3) = 6.3 (4 – 11 x 10^3)
K (mEq/L) = 3.5 (3.5 – 5) Hgb (g/dL) = 16.2 (13.5 – 18 male, 12 – 16 female)
Cl (mEq/L) = 100 (95 – 103) Hct (%) = 48 (38 – 50 male, 36 – 46 female)
HCO3 (mEq/L) = 26 (24 – 30) Plt (cells/mm3) = 246 (150 – 450 x 10^3)
BUN (mg/dL) = 22 (7 – 20) AST (IU/L) = 12 (10 – 40)
SCr (mg/dL) = 1.4 (0.6 – 1.3) ALT (IU/L) = 23 (10 – 40)
Glucose (mg/dL) = 188 (100 – 125) Albumin (g/dL) = 2.9 (3.5 – 5)
Ca (mg/dL) = 8.7 (8.5 – 10.5) A1C (%) = 8.7
Mg (mEq/L) = 1.9 (1.3 – 2.1) PT (sec) = 12 (10 - 13)
PO4 (mg/dL) = 2.5 (2.3 – 4.7) INR = 1

Tests:
CXR: Cardiomegaly, hazy infiltrates bilaterally
ECG: Atrial fibrillation
Plan: Refer to the hospital for management of new onset atrial fibrillation.

Question:
While attempting to manage GH’s heart failure with diuretics, his renal function worsened. His BUN and SCr have increased to 36 and 2.1 respectively. GH’s physician feels that a parenteral anticoagulant is the safest option at this time. Which of the following is a correct recommendation? (Select ALL that apply.)

A. Enoxaparin 70 mg SC daily
B. Enoxaparin 70 mg SC Q12H
C. Heparin 80 units/kg IV bolus, then 18 units/kg/hr IV infusion
D. Heparin 5000 units SC Q8H
E. Desirudin 15 SC Q12

A

A, C. For stroke prevention in atrial fibrillation with CrCl < 30 ml/min, enoxaparin 70 mg SC once daily and the heparin bolus and continuous infusion option listed are both correct recommendations for GH.

98
Q

KP is a 64 year old female who is taking warfarin for atrial fibrillation. She has a past medical history significant for HTN, type 2 diabetes, back pain, and a previous CVA in 2008. KP is a retired nurse and does not drink alcohol or smoke.

Medications:
Prinzide
Lopressor
Coumadin
Glucophage
Victoza
Labs:
Wt: 160 lbs
Ht: 5’1”
SCr: 1.2 mg/dL
KP complains that her back pain is worsening and wants to take Advil or Doan’s for pain relief. The pharmacist should provide the following counseling:

A. It is safe to take either Advil or Doan’s with your other medications. They are both available over the counter.
B. Do not take Advil but it is safe to take Doan’s for pain relief. If symptoms do not improve, please contact your doctor.
C. Do not take Doan’s but it is safe to take Advil for pain relief. If symptoms do not improve, please contact your doctor.
D. It is not safe to take either Advil or Doan’s while on warfarin.
E. You will need to see your doctor for your pain. There are no safe, over the counter options to manage your pain.

A

D. Both ibuprofen (Advil) and magnesium salicylate (Doan’s) are non-steroidal anti-inflammatory medications and are not recommended for use with warfarin due to an increased risk of bleeding. Acetaminophen is the analgesic of choice when a patient is on warfarin.

99
Q

CJ is a 44 y/o female with hyperlipidemia and hypertension. She is referred to a pharmacist-managed anticoagulation clinic with newly diagnosed, first unprovoked episode of lower left leg DVT. She is a teacher and drinks 1 glass of wine per day. Lab/vitals: Weight: 325 lbs, Height: 6’1”, Scr 0.6 mg/dL (1 month ago), Baseline INR 1.1 What INR goal and duration of warfarin therapy should be recommended to treat CJ’s DVT?

A. INR goal of 1.5-2.5 for 6 months
B. INR goal of 2.0-3.0 for 1 year
C. INR goal of 2.0-3.0 for at least 3 months
D. INR goal of 2.0-3.0 indefinitely
E. INR goal of 2.5-3.5 for 3-6 months

A

C. INR goal of 2.0-3.0 for at least 3 months. The 2012 CHEST guidelines recommend to treat the first unprovoked VTE for at least 3 months.

VTE treatment

Provoked: treat for 3 months

Unprovoked: low-to-moderate risk (treat longer than 3 months), high bleeding risk (treat for 3 months)

2 unprovoked VTEs: treat long-term

100
Q

SD had an intracerebral hemorrhage ten days ago secondary to treatment for a cerebrovascular accident. She is considered to be high risk for venous thromboembolism (VTE). What is the best recommendation for preventing VTE in this patient?

A. Unfractionated heparin subcutaneously given BID
B. Aspirin only
C. Low molecular weight heparin given BID
D. Intermittent pneumatic compression devices
E. Apixaban given BID

A

D. Intermittent pneumatic compression devices are recommended for patients with high-bleeding risk. They help keep the blood moving and reduce the risk of clot formation.

101
Q

A 55 year old man (weighing 222 lbs) comes in to the emergency department with crushing chest pain. His creatinine clearance is estimated at 45 mL/min. The physician makes the diagnosis of ST-segment elevation myocardial infarction (STEMI) and wants to initiate enoxaparin right away. What is the correct dose of enoxaparin in this patient?

A. 100 mg SC every 12 hours started immediately
B. 100 mg SC every day sarted immediately
C. 30 mg IV bolus plus 100 mg SC every 12 hours started immediately after the bolus
D. 220 mg SC every day started immediately
E .This patient is not a candidate for enoxaparin therapy

A

C. In patients with STEMI who are less than 75 years old, give enoxaparin 30 mg IV bolus plus a 1 mg/kg SC dose followed by 1 mg/kg SC dose Q12H (max 100 mg for the 1st two doses) in patients with CrCl > 30 mL/min.

enoxaparin (Lovenox): use actual body weight for all. Px VTE (30mg SQ Q12H or 40mg SQdaily; CrCl <30, 30mgSQdaily)

Tx VTE and UA/NSTEMI: 1mg/kg SQ Q12H (or 1.5 mg/kg SC Qdaily for VTE inpatient treatment only; CrCl <30, 1mg/kg SQ daily)

Tx STEMI: For patients <75 y/o, 30mg IV bolus + 1mg/kg SQ dose, then 1 mg/kg SQ Q12H (max 100mg for first 2 doses only); if CrCl <30, 30mg IV bolus + 1mg/kg SQ dose, then 1mg/kg SQ daily. For patients ≥75 y/o, 0.75 mg/kg SQ Q12H (no bolus and max 75mg for first 2 doses only); if CrCl <30, give 1mg/kg SQ daily. For PCI: give 0.3mg/kg IV bolus if last dose was 8-12 hours before balloon inflation.

102
Q

HB has been taking warfarin 6 mg for the past 4 months and all her INR values have been within therapeutic range (INR 2-3) in the past. Today, her INR is 3.2. She has been taking her medication the same way with no additional medications she can recall being recently added to her regimen. What is the most appropriate management of HB’s warfarin therapy at this time?

A. Continue taking warfarin 6 mg daily and test the INR within 1-2 weeks.
B. Hold her warfarin dose until INR is therapeutic, then restart her warfarin back at a dose of 5 mg M, T, Th, F and 6 mg on W, Sat, Sun.
C. Hold her warfarin dose until INR is therapeutic, then restart her warfarin back at a dose of 5 mg W, F and Sunday and 6 mg on T, Th, and Sat. and Monday.
D. Hold her warfarin dose until INR is therapeutic, then restart her warfarin back at a dose of 4 mg daily.
E. Give vitamin K 2.5 mg orally now. Restart warfarin at a dose of 5 mg daily.

A

A.

103
Q

Which of the following anticoagulants can be used to treat patients with prosthetic heart valves?

A. Eliquis
B. Xarelto
C. Apixaban
D. Pradaxa
E. Jantoven

A

E. The newer agents should not be used for patients with prosthetic heart valves. Jantoven is used for treating these patients.

warfarin (Coumadin, Jantoven)

Eliquis (apixaban)

Xarelto (rivaroxaban)

Pradaxa (dabigatran)

104
Q

A nurse practitioner wishes to convert a patient using warfarin to dabigatran. She asks the pharmacist how to manage the conversion. The pharmacist should offer the following advice:

A. Discontinue warfarin and start dabigatran when the INR is below 2.5.
B. Discontinue warfarin and start dabigatran when the INR is at or below 2.
C. Discontinue warfarin and start dabigatran when the INR is at or below 1.5.
D. Stop warfarin and initiate dabigatran the following morning.
E. Discontinue warfarin and start dabigatran when the INR is below 3.

A

B. When converting patients from warfarin therapy to dabigatran, discontinue warfarin and start dabigatran when the international normalized ratio (INR) is below 2.

rivaroxaban (Xarelto): start when INR <3

dabigatran (Pradaxa): start when INR <2

apixaban (Eliquis): start when INR <2

105
Q

The ACCP guidelines recommend a lower warfarin starting dose of less than or equal to 5 mg for the following patient populations: (Select ALL that apply.)

A. Elderly or debilitated
B. Liver disease
C. Heart failure
D. Patients with a high risk of bleeding
E. Those taking medications such as rifampin

A

A, B, C, D, Rifampin is an inducer and requires a much higher dose-not lower.

Dosing: Start 10mg daily for first 2 days in healthy outpatient, then adjust dose per INR values. Doses of ≤5mg may be appropriate starting dose for those who are elderly, malnourished, taking drugs which increases warfarin levels, liver disease, heart failure, or have high risk of bleeding.

106
Q

A pharmacy receives a prescription for warfarin 5 mg PO daily. What is the color of the warfarin tablet?

A. Peach
B. Green
C. Tan
D. Pink
E. Blue

A

A. Warfarin 5 mg comes as a peach tablet.

Warfarin colors: 1mg (pink), 2mg (lavender/purple), 2.5mg (Christmas green), 3mg (tan), 4mg (blue), 5mg (peach), 6mg (forest green/teal), 7.5mg (yellow teeth), 10mg (whiten)

107
Q

BK, a 22 year old female, is admitted for an acute PE. Her past medical history is significant for antithrombin deficiency. Her allergies include cat hair and PCN (shortness of breath for both). Which of the following agents would be best to recommend for treatment of her PE?

A. Heparin
B. Argatroban
C. Lovenox
D. Arixtra
E. Fragmin

A

B. This patient is antithrombin deficient; therefore, heparin, LMWHs and fondaparinux will not work.

108
Q

Which of the following medications will enhance warfarin’s metabolism? (Select ALL that apply.)

A. Rifampin
B. Cimetidine
C. St. John’s Wort
D. Amiodarone
E. Phenytoin

A

A, C, E. Rifampin, St. John’s Wort and phenytoin (among others) will enhance the metabolism of warfarin leading to a decrease in INR.

109
Q

GF is a 54 year old female with a mechanical mitral valve, hypertension, and GERD. What is the correct INR target range for GF?

A. 3.0 - 4.0
B. 2.5 - 3.5
C. 2.0 - 3.0
D. 1.5 - 2.5
E. Warfarin is not indicated for this patient

A

B. Goal INR range for patients with a mechanical mitral valve is 2.5 - 3.5.

Monitor: INR target 2.5, range is 2-3 for most (2.5-3.5 for mechanical mitral valve or 2 mechanical heart valves). Antidote is vitamin K. S-enantiomer is 2.7-3.8 times more potent than R-enantiomer.